Health Asses 4: Restrictive Flashcards

1
Q

Which of the following restrictive lung diseases is primarily associated with environmental occupational hazards?

A) Pleural effusion
B) Kyphoscoliosis
C) Interstitial Pulmonary Fibrosis (IPF)
D) Ankylosing Spondylitis

A

Correct Answer: C) Interstitial Pulmonary Fibrosis (IPF)

Rationale: Interstitial Pulmonary Fibrosis can be caused by occupational exposure to harmful substances such as asbestos, silica, or coal dust. This differentiates it from the other listed conditions, which are not typically associated with occupational exposures. For example, pleural effusion can be secondary to many causes, including infection, malignancy, or heart failure. Kyphoscoliosis and Ankylosing Spondylitis are skeletal disorders that do not have a direct occupational environmental link.

How well did you know this?
1
Not at all
2
3
4
5
Perfectly
2
Q

Which of the following drugs is least likely to be implicated in drug-induced interstitial lung disease?

A) Cyclophosphamide
B) Methotrexate
C) Acetaminophen
D) Bleomycin

A

Correct Answer: C) Acetaminophen

Rationale: The drugs listed in the slide that are known to cause drug-induced interstitial lung disease include Bleomycin, Methotrexate, and Cyclophosphamide. Acetaminophen is commonly used as an analgesic and antipyretic and is not typically associated with interstitial lung disease.

How well did you know this?
1
Not at all
2
3
4
5
Perfectly
3
Q

In the context of restrictive lung disease, severe abdominal distension can primarily affect lung function by:

A) Inducing pleural effusion
B) Impairing chest wall compliance
C) Causing occupational lung diseases
D) Increasing the risk of drug-induced lung patholog

A

Correct Answer: B) Impairing chest wall compliance

Rationale: Severe abdominal distension can lead to elevation of the diaphragm and limit the expansion of the lungs, thus impairing chest wall compliance. This can create a restrictive pattern in lung function testing. The other options, such as pleural effusion or occupational lung diseases, are causes of restrictive lung disease not directly related to abdominal distension.

How well did you know this?
1
Not at all
2
3
4
5
Perfectly
4
Q

A patient diagnosed with a restrictive lung disease due to neuromuscular disorder is most likely to have which underlying condition?

A) Occupational asthma
B) Collagen vascular disease
C) Myasthenia Gravis
D) Silicosis

A

Correct Answer: C) Myasthenia Gravis

“Chest Wall” category
Rationale: Myasthenia Gravis, a neuromuscular disease that can lead to muscle weakness, is listed on the slide under ‘Neuromuscular Disease (Myasthenia/Guillain Barre)’. This disorder can impair respiratory muscle function (CHEST WALL), leading to a restrictive pattern on pulmonary function testing. The other conditions listed do not have a primary neuromuscular component that leads to restrictive lung disease.

How well did you know this?
1
Not at all
2
3
4
5
Perfectly
5
Q

Which of the following is a cause of parenchymal restrictive lung disease that is classified as ‘collagenogenic’?

A) Rheumatoid arthritis
B) Ankylosing Spondylitis
C) Morbid obesity
D) Pneumothorax

A

Correct Answer: A) Rheumatoid arthritis

Rationale: Rheumatoid arthritis is a collagen vascular disease which can cause interstitial lung disease, thus falling into the ‘collagenogenic’ category of parenchymal lung diseases. Ankylosing Spondylitis affects the chest wall and Morbid obesity affects the chest wall compliance, while Pneumothorax is classified under pleural causes of restrictive lung disease.

How well did you know this?
1
Not at all
2
3
4
5
Perfectly
6
Q

What is the primary characteristic of extra-parenchymal restrictive lung diseases as compared to parenchymal restrictive lung diseases?

A) They are characterized by abnormalities in the lung parenchyma itself.
B) They result mainly from pleural diseases and chest wall deformities.
C) They are predominantly caused by occupational exposures.
D) They are usually associated with drug-induced lung pathology.

A

Correct Answer: B) They result mainly from pleural diseases and chest wall deformities.

Rationale: Extra-parenchymal restrictive lung diseases are caused by factors external to the lung parenchyma, such as diseases of the pleura (e.g., pleural effusion, pleural fibrosis) and chest wall deformities (e.g., kyphoscoliosis). In contrast, parenchymal restrictive lung diseases involve the lung tissue itself, including the alveoli, interstitium, blood vessels, and bronchi.

How well did you know this?
1
Not at all
2
3
4
5
Perfectly
7
Q

Among the pleural causes of restrictive lung disease listed, which is most likely to be caused by direct trauma?

A) Pleural thickening
B) Pneumothorax
C) Pleural tumours
D) Pleural effusion

A

Correct Answer: B) Pneumothorax

Rationale: A pneumothorax, which is the presence of air in the pleural space causing lung collapse, can occur due to direct trauma to the chest. Pleural thickening may result from chronic inflammatory conditions, pleural tumours from neoplastic processes, and pleural effusion can have various causes including trauma but is not specifically due to direct trauma.

How well did you know this?
1
Not at all
2
3
4
5
Perfectly
8
Q

Which condition is not typically associated with chest wall-related restrictive lung diseases?

A) Morbid obesity
B) Kyphoscoliosis
C) Pleural tumours
D) Trauma

A

Correct Answer: C) Pleural tumours

Rationale: Chest wall-related restrictive lung diseases are conditions that affect the ability of the chest wall to expand, such as morbid obesity, kyphoscoliosis, and trauma. Pleural tumours, however, are associated with the pleura, not the chest wall, and therefore would not be classified under chest wall-related restrictive lung diseases.

How well did you know this?
1
Not at all
2
3
4
5
Perfectly
9
Q

Ankylosing Spondylitis primarily affects the lung function by:

A) Inducing pleural effusion
B) Reducing chest wall elasticity
C) Causing granulomatous disease
D) Leading to interstitial lung pathology

A

Correct Answer: B) Reducing chest wall elasticity

Rationale: Ankylosing Spondylitis is a form of chronic inflammation of the spine and the sacroiliac joints, which can lead to reduced chest wall elasticity due to stiffening of the spine and rib joints. This condition can result in a restrictive lung disease by limiting the ability of the chest wall to expand during breathing. It does not directly cause granulomatous disease or interstitial lung pathology, and it is not associated with pleural effusion as a primary complication.

How well did you know this?
1
Not at all
2
3
4
5
Perfectly
10
Q

In the context of restrictive lung disease (RLD), what does the abbreviation ‘DLCO’ stand for, and what does it measure?

A) Diffusing Capacity of the Lungs for Carbon Monoxide; it measures the efficiency of gas exchange in the lung alveoli.
B) Diastolic Lung Compliance Output; it measures the compliance of the lung tissue during diastole.
C) Dynamic Lung Capacity Optimization; it measures the ability of the lungs to maximize capacity with exertion.
D) Delayed Lung Clearance Onset; it measures the rate at which lungs clear out pollutants.

A

Correct Answer: A) Diffusing Capacity of the Lungs for Carbon Monoxide; it measures the efficiency of gas exchange in the lung alveoli.

Rationale: The DLCO test assesses the diffusing capacity of the lungs for carbon monoxide and is an important measure in the evaluation of gas exchange in the pulmonary alveoli. In RLD, there is typically a reduction in DLCO due to decreased surface area for gas diffusion.

How well did you know this?
1
Not at all
2
3
4
5
Perfectly
11
Q

Which lung volume is characteristically most reduced in restrictive lung disease (RLD)?

A) Tidal Volume (TV)
B) Total Lung Capacity (TLC)
C) Residual Volume (RV)
D) Expiratory Reserve Volume (ERV)

A

Correct Answer: B) Total Lung Capacity (TLC)

Rationale: In restrictive lung diseases, all lung volumes are typically decreased. However, Total Lung Capacity (TLC) is the most indicative and characteristically reduced volume, as RLD affects the overall expansion and compliance of the lungs.

How well did you know this?
1
Not at all
2
3
4
5
Perfectly
12
Q

Which of the following is a likely consequence of the V/Q mismatch caused by restrictive lung diseases?

A) Increased risk of pulmonary embolism
B) Right-to-left cardiac shunt
C) Hypoxemia
D) Hypercarbia

A

Correct Answer: C) Hypoxemia

Rationale: V/Q mismatch refers to the imbalance between ventilation (air flow) and perfusion (blood flow) in the lungs. In restrictive lung diseases, due to reduced surface area for gas exchange. As lung elasticity worsens, pts become symptomatic d/t hypoxia, inability to clear secretions, and hypoventilation

How well did you know this?
1
Not at all
2
3
4
5
Perfectly
13
Q

The increased FEV1:FVC ratio seen in restrictive lung disease is due to:

A) A proportionally greater reduction in FEV1 compared to FVC.
B) A proportionally greater reduction in FVC compared to FEV1.
C) An increase in both FEV1 and FVC, with a greater increase in FEV1.
D) An increase in both FEV1 and FVC, with a greater increase in FVC.

A

Correct Answer: B) A proportionally greater reduction in FVC compared to FEV1.

Rationale: In restrictive lung disease, both FEV1 (Forced Expiratory Volume in 1 second) and FVC (Forced Vital Capacity) are reduced due to decreased lung compliance. However, FVC is often more affected than FEV1, leading to a normal or increased FEV1:FVC ratio, which is in contrast to obstructive lung diseases where this ratio is decreased.

How well did you know this?
1
Not at all
2
3
4
5
Perfectly
14
Q

A patient with restrictive lung disease is most likely to exhibit which of the following symptoms first?

A) Cyanosis
B) Wheezing
C) Dyspnea on exertion
D) Chronic productive cough

A

Correct Answer: C) Dyspnea on exertion

Rationale: As lung elasticity worsens in restrictive lung disease, the patient’s ability to increase lung volume during physical activity is limited, often leading to dyspnea on exertion as an early symptom. Wheezing is more characteristic of obstructive pulmonary diseases, and chronic productive cough is generally associated with chronic bronchitis. Cyanosis is a late finding, indicative of severe hypoxemia.

How well did you know this?
1
Not at all
2
3
4
5
Perfectly
15
Q

A patient with a Total Lung Capacity (TLC) of 60% of the predicted value is classified as having which severity of restrictive lung disease (RLD)?

A) Mild
B) Moderate
C) Severe
D) Very Severe

A

Correct Answer: B) Moderate

Rationale: Based on the classification provided, a TLC of 60% of the predicted value falls into the category of moderate restrictive lung disease, which is defined as a TLC between 50-65% of the predicted value.

How well did you know this?
1
Not at all
2
3
4
5
Perfectly
16
Q

A patient’s pulmonary function test reveals a Total Lung Capacity (TLC) of 70% of the predicted value. This indicates:

A) The patient does not have restrictive lung disease.
B) The patient has mild restrictive lung disease.
C) The patient has a normal lung function.
D) The patient’s condition is inconsistent with restrictive lung disease.

A

Correct Answer: B) The patient has mild restrictive lung disease.

Rationale: According to the slide, a TLC that ranges from 65-80% of the predicted value is indicative of mild restrictive lung disease. Therefore, a TLC of 70% would fall within this range.

How well did you know this?
1
Not at all
2
3
4
5
Perfectly
17
Q

Which condition listed under ‘Acute Intrinsic Restrictive Lung Disease’ is most closely associated with a rapid onset following direct lung injury?

A) Aspiration
B) Neurogenic problems
C) High altitude
D) Upper airway obstruction

A

Correct Answer: A) Aspiration

Rationale: Aspiration refers to the inhalation of foreign material into the lungs, which can lead to direct lung injury and acute pulmonary edema. It often presents with a rapid onset following the aspiration event. The other conditions listed, such as neurogenic problems or high altitude, may lead to acute lung issues but are not primarily due to direct lung injury.

How well did you know this?
1
Not at all
2
3
4
5
Perfectly
18
Q

‘Guillain-Barré syndrome’ is categorized under which of the following in the provided list?

A) Acute Intrinsic Restrictive Lung Disease
B) Chronic Intrinsic Restrictive Lung Disease
C) Disorders of the Chest Wall, Pleura, and Mediastinum
D) Other

A

Correct Answer: C) Disorders of the Chest Wall, Pleura, and Mediastinum

Rationale: Guillain-Barré syndrome is listed under the category of ‘Neuromuscular disorders’ which falls under the broader classification of ‘Disorders of the Chest Wall, Pleura, and Mediastinum.’ It is a disorder that affects the peripheral nerves and can impact the muscles of respiration, leading to restrictive lung disease.

How well did you know this?
1
Not at all
2
3
4
5
Perfectly
19
Q

Which of the following is not a chronic intrinsic cause of restrictive lung disease?

A) Sarcoidosis
B) Hypersensitivity pneumonitis
C) Pneumothorax
D) Eosinophilic granuloma

A

Correct Answer: C) Pneumothorax

Rationale: Pneumothorax, which is the presence of air in the pleural space, is classified under ‘Disorders of the Chest Wall, Pleura, and Mediastinum’ and not as a chronic intrinsic cause of restrictive lung disease. Sarcoidosis, hypersensitivity pneumonitis, and eosinophilic granuloma are all chronic intrinsic conditions that can lead to restrictive lung patterns.

How well did you know this?
1
Not at all
2
3
4
5
Perfectly
19
Q

Which condition among the following is typically not associated with ‘Chronic Intrinsic Restrictive Lung Disease’?

A) Alveolar proteinosis
B) Muscular dystrophies
C) Lymphangioleiomyomatosis
D) Drug-induced pulmonary fibrosis

A

Correct Answer: B) Muscular dystrophies

Rationale: Muscular dystrophies are listed under ‘Chest Wall’ because they are primarily neuromuscular disorders that can lead to restrictive lung disease due to weakened respiratory muscles, rather than being intrinsic to the lung parenchyma. Alveolar proteinosis, lymphangioleiomyomatosis, and drug-induced pulmonary fibrosis are all intrinsic to the lung tissue and cause chronic restrictive lung disease.

How well did you know this?
1
Not at all
2
3
4
5
Perfectly
20
Q

The presence of which of the following conditions would most likely suggest a diagnosis of acute rather than chronic restrictive lung disease?

A) Upper airway obstruction (negative pressure)
B) Drug-induced pulmonary fibrosis
C) Obesity
D) Sarcoidosis

A

Correct Answer: A) Upper airway obstruction (negative pressure)

Rationale: Upper airway obstruction due to negative pressure is an acute event that can lead to restrictive lung disease due to the inability to generate sufficient negative pressure to inflate the lungs properly. In contrast, drug-induced pulmonary fibrosis, obesity, and sarcoidosis are associated with chronic processes leading to restrictive lung disease.

How well did you know this?
1
Not at all
2
3
4
5
Perfectly
21
Q

Which imaging modality is traditionally used to identify the characteristic findings of pulmonary edema mentioned in the text?

A) Computerized Tomography (CT) scan
B) Magnetic Resonance Imaging (MRI)
C) Chest X-Ray (CXR)
D) Positron Emission Tomography (PET) scan

A

Correct Answer: C) Chest X-Ray (CXR)

Rationale: Pulmonary edema is traditionally identified by bilateral, symmetric perihilar opacities on a Chest X-Ray (CXR). The text specifically refers to CXR as the imaging modality used to recognize the appearance of pulmonary edema.

How well did you know this?
1
Not at all
2
3
4
5
Perfectly
22
Q

In the context of Acute Respiratory Distress Syndrome (ARDS), what pathological finding is commonly present?

A) Cardiogenic intravascular fluid retention
B) Unilateral pulmonary opacities
C) Diffuse alveolar damage
D) Isolated pleural effusion

A

Correct Answer: C) Diffuse alveolar damage

Rationale: Diffuse alveolar damage is a common finding in the lungs of patients with ARDS and is associated with increased-permeability pulmonary edema. This is a hallmark of ARDS and indicates severe injury to the alveolar-capillary barrier.

How well did you know this?
1
Not at all
2
3
4
5
Perfectly
23
Q

The pulmonary edema fluid characterized by increased capillary permeability is likely to contain:

A) A low concentration of protein and few secretory products
B) A normal concentration of protein typical of plasma
C) A high concentration of protein and secretory products
D) Only secretory products without protein

A

Correct Answer: C) A high concentration of protein and secretory products

Rationale: Pulmonary edema that is due to increased capillary permeability, such as in non-cardiogenic edema, is characterized by a high concentration of protein and secretory products. This distinguishes it from cardiogenic pulmonary edema, which typically has a lower protein concentration.

How well did you know this?
1
Not at all
2
3
4
5
Perfectly
24
Q

What pathophysiologic process in pulmonary edema leads to ‘capillary stress failure’?

A) Decreased surfactant production by alveolar cells
B) Collapse of the alveolar air spaces
C) Mechanical or ischemic disruption of the alveolar-capillary barrier
D) Excessive mucous secretion in the bronchial passages

A

Correct Answer: C) Mechanical or ischemic disruption of the alveolar-capillary barrier

Rationale: ‘Capillary stress failure’ refers to the disruption of the alveolar-capillary barrier due to mechanical or ischemic stress. This barrier is crucial for maintaining fluid homeostasis in the lungs. When it is compromised, “fluid leaks into the interstitial and alveolar spaces”, leading to pulmonary edema.

How well did you know this?
1
Not at all
2
3
4
5
Perfectly
25
Q

What clinical finding is strongly associated with pulmonary edema due to increased capillary pressure?

A) Bilateral wheezing on auscultation
B) A pleural rub heard during the respiratory cycle
C) Bilateral symmetric perihilar opacities on imaging
D) Unilateral absence of breath sounds

A

Correct Answer: C) Bilateral symmetric perihilar opacities on imaging

Rationale: Bilateral symmetric perihilar opacities on a chest X-ray are typical findings in pulmonary edema, especially when caused by increased capillary pressure. This radiographic appearance reflects the accumulation of fluid around the central areas of the lungs adjacent to the hilum.

How well did you know this?
1
Not at all
2
3
4
5
Perfectly
26
Q

The edema fluid in pulmonary edema due to ARDS typically differs from cardiogenic pulmonary edema in which way?

A) It has a higher concentration of red blood cells.
B) It has a lower concentration of white blood cells.
C) It has a higher concentration of protein and secretory products.
D) It is typically more serous and less viscous.

A

Correct Answer: C) It has a higher concentration of protein and secretory products.

Rationale: Pulmonary edema fluid due to ARDS, a type of non-cardiogenic pulmonary edema, is characterized by increased capillary permeability, which leads to a high concentration of protein and secretory products in the edema fluid. This is a distinguishing feature from cardiogenic pulmonary edema, which generally has a lower protein content in the fluid.

How well did you know this?
1
Not at all
2
3
4
5
Perfectly
27
Q

How has bedside lung ultrasound altered the approach to diagnosing pulmonary edema?

A) It has replaced the need for chest X-ray imaging entirely.
B) It offers a rapid, non-invasive means to support the diagnosis at the patient’s bedside.
C) It has limited applicability due to the need for high technical expertise.
D) It is only used when traditional imaging methods are inconclusive.

A

Correct Answer: B) It offers a rapid, non-invasive means to support the diagnosis at the patient’s bedside.

Rationale: Bedside lung ultrasound has emerged as a valuable tool in the diagnosis of pulmonary edema, providing a rapid, non-invasive means to detect fluid accumulation in the lungs at the patient’s bedside. While it has not replaced traditional imaging methods like CXR or CT scans, it serves as a complementary tool, especially when immediate information is required.

How well did you know this?
1
Not at all
2
3
4
5
Perfectly
28
Q

In cardiogenic pulmonary edema, activation of which system contributes to the clinical symptoms?

A) Parasympathetic nervous system
B) Sympathetic nervous system
C) Renin-angiotensin-aldosterone system
D) Peripheral nervous system

A

Correct Answer: B) Sympathetic nervous system

Rationale: Cardiogenic pulmonary edema is characterized by symptoms such as marked dyspnea, tachypnea, and elevated cardiac pressures. These symptoms are exacerbated by the activation of the Sympathetic Nervous System (SNS), which is more pronounced in cardiogenic compared to non-cardiogenic pulmonary edema.

How well did you know this?
1
Not at all
2
3
4
5
Perfectly
29
Q

Cardiogenic pulmonary edema should be suspected in a patient with decreased cardiac function and which of the following conditions?

A) Chronic hypertension
B) Acute aortic regurgitation
C) Chronic obstructive pulmonary disease
D) Pulmonary embolism

A

Correct Answer: B) Acute aortic regurgitation

Rationale: The risk of cardiogenic pulmonary edema is increased with conditions that acutely increase preload, such as acute aortic regurgitation and acute mitral valve regurgitation. These conditions can lead to sudden volume overload in the left ventricle, contributing to the development of pulmonary edema.

How well did you know this?
1
Not at all
2
3
4
5
Perfectly
30
Q

What condition primarily increases afterload, leading to a higher risk of cardiogenic pulmonary edema?

A) Mitral valve prolapse
B) Atrial septal defect
C) Left ventricular outflow tract (LVOT) obstruction
D) Tricuspid regurgitation

A

Correct Answer: C) Left ventricular outflow tract (LVOT) obstruction

Rationale: Conditions that increase afterload, such as Left Ventricular Outflow Tract (LVOT) obstruction, mitral stenosis, and renovascular hypertension, can lead to an increased risk of cardiogenic pulmonary edema. LVOT obstruction increases the resistance against which the heart must pump, leading to elevated cardiac pressures and contributing to pulmonary edema.

How well did you know this?
1
Not at all
2
3
4
5
Perfectly
31
Q

When evaluating a patient for possible cardiogenic pulmonary edema, which diagnostic finding would corroborate decreased cardiac function as a potential cause?

A) Elevated peak expiratory flow rate
B) Diminished ejection fraction on echocardiography
C) Increased forced vital capacity on spirometry
D) Broadened QRS complex on electrocardiogram

A

Correct Answer: B) Diminished ejection fraction on echocardiography

Rationale: A decreased ejection fraction observed on echocardiography indicates diminished systolic cardiac function, which can be a contributing factor to cardiogenic pulmonary edema. Echocardiography is a key diagnostic tool for assessing cardiac function, including both systolic and diastolic function. Reduced ejection fraction specifically points to decreased systolic performance, which is when the heart has difficulty effectively pumping blood out into the systemic circulation, potentially leading to pulmonary congestion and edema.

How well did you know this?
1
Not at all
2
3
4
5
Perfectly
32
Q

Which of the following is NOT a consistent clinical presentation of the initial sx of cardiogenic pulmonary edema due to compensatory mechanisms?

A) Hypotension
B) Tachypnea
C) Elevated cardiac pressures
D) Marked dyspnea

A

Correct Answer: A) Hypotension

Rationale: In cardiogenic pulmonary edema, blood pressure may be maintained at normal or even elevated levels initially due to compensatory mechanisms, such as activation of the sympathetic nervous system and the renin-angiotensin-aldosterone system, despite decreased cardiac function. Over time, if the heart failure progresses and these mechanisms are overwhelmed, blood pressure may then fall. Therefore, the presence of normal or elevated blood pressure does not rule out cardiogenic pulmonary edema, particularly in the early compensatory stage of heart failure. Tachypnea, elevated cardiac pressures, and marked dyspnea are more consistent features of cardiogenic pulmonary edema.

How well did you know this?
1
Not at all
2
3
4
5
Perfectly
33
Q

What is the primary mechanism by which negative pressure pulmonary edema (NPPE) develops after acute upper airway obstruction is relieved?

A) Increased capillary hydrostatic pressure due to positive pressure ventilation
B) Increased left ventricular preload and afterload due to intense inspiratory efforts against an obstruction
C) Decreased pulmonary interstitial hydrostatic pressure due to systemic vasodilation
D) Decreased venous return due to high intrathoracic pressures

A

Correct Answer: B) Increased left ventricular preload and afterload due to intense inspiratory efforts against an obstruction

Rationale: Negative pressure pulmonary edema develops when there is a sudden relief from an upper airway obstruction. The vigorous inspiratory efforts against the obstruction lead to a decrease in intrapleural pressure which in turn increases the venous return to the heart (preload) and increases the left ventricular afterload. The negative intrathoracic pressure generated also increases the transmural pressure gradient across the pulmonary capillary bed, promoting the transudation of fluid into the alveolar and interstitial spaces.

How well did you know this?
1
Not at all
2
3
4
5
Perfectly
34
Q

Which of the following conditions is least likely to be a cause of negative pressure pulmonary edema (NPPE)?

A) Obstructive Sleep Apnea (OSA)
B) Bronchial asthma
C) Laryngospasm
D) Epiglottitis

A

Correct Answer: B) Bronchial asthma

Rationale: NPPE is specifically associated with the relief of an acute upper airway obstruction, which includes conditions like laryngospasm, epiglottitis, and obstructive sleep apnea (OSA), where the obstruction is at the level of the upper airway. Bronchial asthma is a lower airway disorder characterized by bronchoconstriction, inflammation, and increased mucus production, and is not a common cause of NPPE.

How well did you know this?
1
Not at all
2
3
4
5
Perfectly
35
Q

In negative pressure pulmonary edema, the intense activation of which body system contributes to the pathophysiology?

A) Gastrointestinal system due to reflux and aspiration
B) Sympathetic nervous system due to stress response
C) Urinary system due to alterations in renal perfusion
D) Endocrine system due to hormonal dysregulation

A

Correct Answer: B) Sympathetic nervous system due to stress response

Rationale: NPPE is characterized by intense sympathetic nervous system (SNS) activation as a result of the stress response to acute airway obstruction and negative intrathoracic pressures. This leads to hypertension (HTN) and central displacement of blood volume, which are contributory factors in the development of pulmonary edema.

How well did you know this?
1
Not at all
2
3
4
5
Perfectly
36
Q

What time frame is commonly observed for the onset of pulmonary edema following the relief of an obstruction in NPPE?

A) Within a few seconds to minutes
B) A few minutes to 2-3 hours
C) 12-24 hours after the obstruction is relieved
D) Several days post-obstruction relief

A

Correct Answer: B) A few minutes to 2-3 hours

Rationale: The onset of NPPE can vary, but typically it ranges from a few minutes to 2-3 hours after the relief of the upper airway obstruction. It is an acute event that follows the pathophysiological sequence of relief from the obstruction, increased venous return, increased left ventricular afterload, and finally the transudation of fluid into the lungs.

How well did you know this?
1
Not at all
2
3
4
5
Perfectly
37
Q

What role does the decrease in intrapleural pressure play in the pathophysiology of negative pressure pulmonary edema?

A) It leads to a reduction in the alveolar surface area for gas exchange.
B) It increases the interstitial hydrostatic pressure, preventing fluid extravasation.
C) It causes a decrease in venous return, reducing cardiac output.
D) It results in increased venous return and left ventricular afterload, contributing to pulmonary edema.

A

Correct Answer: D) It results in increased venous return and left ventricular afterload, contributing to pulmonary edema.

Rationale: In the setting of negative pressure pulmonary edema, a significant decrease in intrapleural pressure, particularly due to forceful inspiratory efforts against an obstructed airway, enhances venous return to the right side of the heart. This increased preload, along with the negative pressure exerted on the lung vasculature, leads to an increased left ventricular afterload. The augmented afterload, coupled with the stress response-induced hypertension, intensifies the pressure in the left side of the heart. These hemodynamic changes, together with the negative pressure’s effect on the pulmonary capillaries, facilitate the translocation of fluid from the capillaries into the interstitial and alveolar spaces, resulting in pulmonary edema.

How well did you know this?
1
Not at all
2
3
4
5
Perfectly
38
Q

For the treatment of Negative Pressure Pulmonary Edema (NPPE), what is the mainstay of therapy?

A) Administration of corticosteroids
B) Providing supplemental oxygen and ensuring airway patency
C) Immediate intubation and prolonged mechanical ventilation
D) Diuretics and fluid restriction

A

Correct Answer: B) Providing supplemental oxygen and ensuring airway patency

Rationale: The mainstay of treatment for NPPE involves providing supplemental oxygen to address hypoxemia and ensuring that the airway is patent to facilitate adequate ventilation. NPPE is usually self-limited, and these supportive measures are typically sufficient while the edema resolves spontaneously.

How well did you know this?
1
Not at all
2
3
4
5
Perfectly
39
Q

How long does it typically take for radiographic evidence of NPPE to resolve after treatment initiation?

A) Within 1-2 hours
B) Within 12-24 hours
C) 3-5 days post-treatment
D) Up to a week or more

A

Correct Answer: B) Within 12-24 hours

Rationale: The radiographic evidence of NPPE generally resolves relatively quickly, usually within 12-24 hours after the episode. This resolution coincides with the self-limiting nature of the condition and the effectiveness of supportive care including oxygen therapy and ensuring a patent airway.

How well did you know this?
1
Not at all
2
3
4
5
Perfectly
40
Q

Mechanical ventilation is employed in the treatment of NPPE under what circumstances?

A) As a preventive measure in all cases of NPPE
B) Routinely, immediately upon diagnosis
C) When supplemental oxygen and a patent airway do not suffice
D) Only after radiographic evidence fails to resolve in 24 hours

A

Correct Answer: C) When supplemental oxygen and a patent airway do not suffice

Rationale: Mechanical ventilation may be required in the treatment of NPPE if the initial management with supplemental oxygen and maintaining a patent airway is not effective in relieving hypoxemia and respiratory distress. It is not used preventively in all cases nor routinely upon diagnosis but is reserved for cases where conservative measures are inadequate.

How well did you know this?
1
Not at all
2
3
4
5
Perfectly
41
Q

What is the primary mechanism by which neurogenic pulmonary edema develops following an acute CNS injury?

A) Altered pulmonary capillary permeability secondary to infection
B) Generalized vasoconstriction and blood volume shift into the pulmonary circulation due to sympathetic nervous system discharge
C) Primary failure of the left side of the heart leading to back-pressure
D) Chronic elevation of pulmonary venous pressure due to renal artery stenosis

A

Correct Answer: B) Generalized vasoconstriction and blood volume shift into the pulmonary circulation due to sympathetic nervous system discharge

Rationale: Neurogenic pulmonary edema is caused by a massive sympathetic discharge following an acute CNS injury. This outpouring of sympathetic impulses leads to generalized vasoconstriction and a consequent shift of blood volume into the pulmonary circulation, increasing pulmonary capillary pressure, and promoting the translocation of fluid into the pulmonary interstitium and alveoli.

How well did you know this?
1
Not at all
2
3
4
5
Perfectly
42
Q

In what timeframe does neurogenic pulmonary edema typically manifest after a central nervous system (CNS) injury?

A) 1-2 weeks following the injury
B) 24-48 hours after the injury
C) Minutes to hours after the injury
D) Immediately at the moment of injury

A

Correct Answer: C) Minutes to hours after the injury

Rationale: Neurogenic pulmonary edema can develop rapidly, often within minutes to hours after an acute CNS injury. It may present during the perioperative period, which is a critical time when the patient is under close observation, and any acute changes in respiratory function can be readily noted and addressed.

How well did you know this?
1
Not at all
2
3
4
5
Perfectly
43
Q

Beyond fluid translocation, what other direct consequence can result from the pathophysiological changes of neurogenic pulmonary edema?

A) Decrease in pulmonary compliance due to fibrotic changes
B) Injury to pulmonary blood vessels due to pulmonary hypertension and hypervolemia
C) Obstruction of the upper airways due to laryngeal edema
D) Increased risk of pneumonia due to aspiration

A

Correct Answer: B) Injury to pulmonary blood vessels due to pulmonary hypertension and hypervolemia

Rationale: The increased pulmonary capillary pressure and blood volume within the pulmonary circulation, secondary to intense sympathetic nervous system activation in neurogenic pulmonary edema, can lead to pulmonary hypertension and hypervolemia. These changes increase the stress on pulmonary blood vessels, which can result in injury to the vessel walls.

How well did you know this?
1
Not at all
2
3
4
5
Perfectly
44
Q

What is a key risk factor for the development of re-expansion pulmonary edema (REPE)?

A) A gradual, controlled expansion of a collapsed lung
B) Presence of less than 1 liter of air or fluid in the pleural space
C) Duration of lung collapse less than 24 hours
D) Rapid re-expansion of a collapsed lung

A

Correct Answer: D) Rapid re-expansion of a collapsed lung

Rationale: One of the key risk factors for REPE is the rapid re-expansion of a collapsed lung. This can cause a sudden shift in pressures within the thoracic cavity and lead to the development of pulmonary edema. A more controlled re-expansion is less likely to cause REPE.

How well did you know this?
1
Not at all
2
3
4
5
Perfectly
45
Q

What does the presence of a high protein content in pulmonary edema fluid indicate about the pathophysiology of REPE?

A) Decreased lymphatic drainage from the lung parenchyma
B) Increased capillary hydrostatic pressure
C) Enhanced capillary membrane permeability
D) Reduction in plasma oncotic pressure

A

Correct Answer: C) Enhanced capillary membrane permeability

Rationale: The high protein content in the edema fluid of REPE suggests that the underlying mechanism involves enhanced permeability of the pulmonary capillary membrane, allowing proteins to pass from the capillaries into the alveolar space.

How well did you know this?
1
Not at all
2
3
4
5
Perfectly
46
Q

Regarding the supportive care treatment for REPE, which of the following interventions is typically included?

A) Aggressive diuretic therapy
B) Supplemental oxygen and careful fluid management
C) Immediate surgical intervention
D) Systemic administration of vasodilators

A

Correct Answer: B) Supplemental oxygen and careful fluid management

Rationale: Supportive care for REPE typically includes supplemental oxygen to maintain adequate oxygenation and careful fluid management to prevent further exacerbation of pulmonary edema. It aims to support the patient’s respiratory function while the condition resolves spontaneously.

How well did you know this?
1
Not at all
2
3
4
5
Perfectly
47
Q

Which factor increases the risk of REPE after a pneumothorax or pleural effusion is resolved?

A) Smaller volume of air or fluid in the pleural space
B) Shorter duration of lung collapse
C) Slower rate of lung re-expansion
D) Larger volume of air or fluid in the pleural space

A

Correct Answer: D) Larger volume of air or fluid in the pleural space

Rationale: A larger volume of air or liquid in the pleural space (>1 liter) increases the risk of developing REPE when the lung is re-expanded. The pleural space’s capacity to accommodate large volumes can lead to significant changes in intrapleural pressure during re-expansion.

How well did you know this?
1
Not at all
2
3
4
5
Perfectly
48
Q

The duration of lung collapse considered to increase the risk for REPE is:

A) Less than 12 hours
B) More than 24 hours
C) Between 12 and 24 hours
D) Any duration of lung collapse

A

Correct Answer: B) More than 24 hours

Rationale: The risk of REPE is increased if the lung has been collapsed for an extended period, particularly more than 24 hours. Longer durations of collapse can lead to alterations in lung and pleural space physiology that contribute to the risk of edema upon re-expansion.

How well did you know this?
1
Not at all
2
3
4
5
Perfectly
49
Q

What is indicated by the high protein concentration in pulmonary edema fluid caused by drug use?

A) Cardiogenic etiology of the pulmonary edema
B) Increased capillary permeability contributing to the edema
C) Renal origin of the pulmonary edema
D) Low oncotic pressure in the pulmonary circulation

A

Correct Answer: B) Increased capillary permeability contributing to the edema

Rationale: A high protein concentration in pulmonary edema fluid is indicative of increased capillary permeability, also known as non-cardiogenic or high-permeability pulmonary edema. This type of edema is characteristic of drug-induced pulmonary edema rather than cardiogenic causes, which usually result in a transudative edema with lower protein content.

How well did you know this?
1
Not at all
2
3
4
5
Perfectly
50
Q

Cocaine-induced pulmonary edema is associated with what additional cardiovascular effect?

A) Bradycardia
B) Pulmonary vasodilation
C) Pulmonary vasoconstriction and myocardial ischemia
D) Reduction of systemic vascular resistance

A

Correct Answer: C) Pulmonary vasoconstriction and myocardial ischemia

Rationale: Cocaine can induce pulmonary edema through its effects on the cardiovascular system, which include causing pulmonary vasoconstriction and the potential to induce acute myocardial ischemia. These effects contribute to the development of pulmonary edema.

How well did you know this?
1
Not at all
2
3
4
5
Perfectly
51
Q

Why is naloxone not effective in reversing opioid-induced pulmonary edema?

A) It only reverses the sedative effects of opioids, not the respiratory or cardiovascular effects.
B) It increases pulmonary vascular permeability.
C) It cannot cross the blood-brain barrier.
D) Opioid-induced pulmonary edema is not mediated by opioid receptors.

A

Correct Answer: D) Opioid-induced pulmonary edema is not mediated by opioid receptors.

Rationale: Naloxone is an opioid antagonist that reverses the central nervous system effects of opioids, including respiratory depression. However, opioid-induced pulmonary edema is a non-receptor-mediated effect. Therefore, naloxone does not reverse the pulmonary complications associated with opioid overdose.

How well did you know this?
1
Not at all
2
3
4
5
Perfectly
52
Q

What is a likely treatment approach for a patient with drug-induced pulmonary edema who is experiencing respiratory distress?

A) Oral corticosteroids
B) Intubation and mechanical ventilation
C) High-dose beta-blockers
D) Immediate dialysis

A

Correct Answer: B) Intubation and mechanical ventilation

Rationale: Supportive care, including intubation and mechanical ventilation, may be necessary for the treatment of drug-induced pulmonary edema, especially if the patient is in respiratory distress. This helps maintain adequate oxygenation and ventilation while the effects of the drug subside and the edema resolves.

How well did you know this?
1
Not at all
2
3
4
5
Perfectly
53
Q

Diffuse alveolar hemorrhage (DAH) should be considered when a patient with pulmonary edema does not respond to which type of treatment?

A) Diuretics
B) Antibiotics
C) Antivirals
D) Antifungals

A

Correct Answer: A) Diuretics

Rationale: If a patient’s pulmonary edema is refractory to diuretic treatment, it suggests that the edema may not be of cardiogenic origin. Diffuse alveolar hemorrhage (DAH) is a condition that presents with similar symptoms to pulmonary edema but involves bleeding into the alveoli. It is not typically responsive to diuretics, as the issue is related to bleeding rather than fluid overload.

How well did you know this?
1
Not at all
2
3
4
5
Perfectly
54
Q

High-Altitude Pulmonary Edema (HAPE) is most likely to occur at what minimum altitude?

A) 1500m
B) 2500m
C) 3500m
D) 4500m

A

Correct Answer: B) 2500m

Rationale: HAPE can occur at altitudes typically ranging from 2500 to 5000 meters above sea level. The risk increases with the rate of ascent to high altitude and with higher altitudes.

How well did you know this?
1
Not at all
2
3
4
5
Perfectly
55
Q

The pathophysiology of HAPE involves which primary mechanism?

A) Allergic reactions to decreased atmospheric pressure
B) Hypoxic pulmonary vasoconstriction leading to increased pulmonary vascular pressure
C) Bacterial infection secondary to immune system suppression at high altitude
D) Overhydration due to excessive fluid intake at high altitudes

A

Correct Answer: B) Hypoxic pulmonary vasoconstriction leading to increased pulmonary vascular pressure

Rationale: The primary mechanism thought to cause HAPE is hypoxic pulmonary vasoconstriction in response to the lower oxygen levels at high altitudes. This hypoxia-induced response increases pulmonary vascular pressure and can lead to a high-permeability type of pulmonary edema.

How well did you know this?
1
Not at all
2
3
4
5
Perfectly
56
Q

What is the typical time frame for the onset of HAPE after reaching a high altitude?

A) Within the first 6-12 hours
B) 12-24 hours
C) 24-48 hours
D) 48-72 hours

A

Correct Answer: D) 48-72 hours

Rationale: HAPE usually develops gradually and typically manifests within 48-72 hours after arrival at a high altitude.

How well did you know this?
1
Not at all
2
3
4
5
Perfectly
57
Q

Which treatment is indicated for immediate management of HAPE?

A) Supplemental oxygen and rapid ascent to a higher altitude for acclimatization
B) Diuretics and maintenance at the current altitude
C) Supplemental oxygen and rapid descent to a lower altitude
D) Antibiotic therapy and continuation of physical exertion

A

Correct Answer: C) Supplemental oxygen and rapid descent to a lower altitude

Rationale: Immediate treatment for HAPE includes administration of supplemental oxygen and a rapid descent to a lower altitude, which are critical for reducing hypoxia and pulmonary arterial pressure, thereby alleviating symptoms.

How well did you know this?
1
Not at all
2
3
4
5
Perfectly
58
Q

What adjunct therapy may improve oxygenation in a patient with HAPE?

A) Beta-2 agonists
B) Inhalation of nitric oxide
C) Oral corticosteroids
D) Antihistamines

A

Correct Answer: B) Inhalation of nitric oxide

Rationale: Inhalation of nitric oxide is a treatment option that may improve oxygenation in HAPE by causing vasodilation in the pulmonary vasculature, thus reducing pulmonary hypertension and aiding in the relief of edema

How well did you know this?
1
Not at all
2
3
4
5
Perfectly
59
Q

In patients with pulmonary edema, elective surgery should be approached with which of the following considerations?

A) Proceeding as planned to avoid delay
B) Postponement until cardiorespiratory function is optimized
C) Immediate surgery to resolve the pulmonary edema
D) Surgery without any special preparations

A

Correct Answer: B) Postponement until cardiorespiratory function is optimized

Rationale: Elective surgery should be delayed in patients with pulmonary edema, and efforts should be made to optimize cardiorespiratory function before proceeding with surgery. This helps to minimize perioperative risks and complications related to the edema.

How well did you know this?
1
Not at all
2
3
4
5
Perfectly
60
Q

What ventilatory strategy is indicated for patients with pulmonary edema under anesthesia?

A) High tidal volume (TV) ventilation to ensure adequate oxygenation
B) Low tidal volume (TV) ventilation and a respiratory rate (RR) of 14-18 breaths per minute
C) High-frequency ventilation to maximize oxygen delivery
D) Low respiratory rate (RR) to minimize oxygen consumption

A

Correct Answer: B) Low tidal volume (TV) ventilation and a respiratory rate (RR) of 14-18 breaths per minute

Rationale: Current evidence supports the benefit of ventilation using low tidal volume and a respiratory rate of 14-18 while keeping end-inspiratory plateau pressures below 30 cm H2O. This approach can help to prevent additional lung injury caused by mechanical ventilation, such as ventilator-induced lung injury (VILI).

How well did you know this?
1
Not at all
2
3
4
5
Perfectly
61
Q

Which monitoring is particularly useful in the assessment and treatment of pulmonary edema in the perioperative setting?

A) Neurological monitoring with EEG
B) Hemodynamic monitoring
C) Continuous temperature monitoring
D) Intraoperative glucose monitoring

A

Correct Answer: B) Hemodynamic monitoring

Rationale: Hemodynamic monitoring is particularly useful in the assessment and treatment of pulmonary edema, as it helps in understanding the fluid status and cardiac function of the patient, thereby guiding appropriate fluid management and cardiovascular support.

How well did you know this?
1
Not at all
2
3
4
5
Perfectly
62
Q

What does the recommendation to carefully titrate PEEP with an inspiratory pause aim to achieve in patients with pulmonary edema?

A) Reduce the respiratory rate to avoid hyperventilation
B) Optimize lung compliance and oxygenation
C) Increase cardiac preload and improve cardiac output
D) Minimize the risk of nosocomial infections

A

Correct Answer: B) Optimize lung compliance and oxygenation

Rationale: The careful titration of positive end-expiratory pressure (PEEP) along with an inspiratory pause is recommended to optimize lung compliance, which can be reduced in pulmonary edema. This helps to enhance oxygenation while minimizing potential ventilator-induced lung injury.

How well did you know this?
1
Not at all
2
3
4
5
Perfectly
63
Q

In the context of extubation criteria, why should tachypnea not be the sole determinant in patients with restrictive lung disease (RLD)?

A) Patients with RLD can have rapid, shallow breathing as a baseline.
B) Tachypnea indicates improved lung function and readiness for extubation.
C) Tachypnea is a normal response to anesthesia and surgery.
D) Tachypnea is unrelated to lung diseases and is purely a cardiac symptom.

A

Correct Answer: A) Patients with RLD can have rapid, shallow breathing as a baseline.

Rationale: Patients with restrictive lung disease often present with rapid, shallow breathing as a baseline respiratory pattern due to their decreased lung compliance. Therefore, tachypnea should not be used as the sole criterion for delaying extubation if gas exchange and other assessments are satisfactory.

How well did you know this?
1
Not at all
2
3
4
5
Perfectly
64
Q

Which patients are particularly at risk for developing chemical pneumonitis?

A) Patients with heightened airway reflexes
B) Patients with decreased airway reflexes
C) Patients with a history of chronic obstructive pulmonary disease
D) Patients with pre-existing pulmonary hypertension

A

Correct Answer: B) Patients with decreased airway reflexes

Rationale: Patients with decreased airway reflexes are at an increased risk for aspiration, which can lead to chemical pneumonitis. These reflexes are critical for protecting the airway during swallowing and other times when foreign material could enter the respiratory tract.

How well did you know this?
1
Not at all
2
3
4
5
Perfectly
65
Q

What is the purpose of elevating the head of the bed (HOB) during intubation and extubation?

A) To improve venous return and cardiac output
B) To decrease the risk of ventilator-associated pneumonia
C) To reduce the risk of aspiration
D) To enhance patient comfort and reduce anxiety

A

Correct Answer: C) To reduce the risk of aspiration

Rationale: Elevating the head of the bed (HOB) during intubation and extubation is a maneuver to decrease the risk of aspiration. By using gravity, this position helps prevent gastric contents from entering the respiratory tract if regurgitation occurs.

How well did you know this?
1
Not at all
2
3
4
5
Perfectly
66
Q

Which of the following is a common presenting symptom of chemical pneumonitis?

A) Gradual onset of wheezing
B) Abrupt onset of dyspnea, tachycardia, and desaturation
C) Slowly progressive cough with sputum production
D) Chronic shortness of breath with exertion

A

Correct Answer: B) Abrupt onset of dyspnea, tachycardia, and desaturation

Rationale: Chemical pneumonitis usually presents with an abrupt onset of symptoms such as dyspnea, tachycardia, and desaturation due to the acute inflammatory response in the lungs after aspiration of gastric contents.

How well did you know this?
1
Not at all
2
3
4
5
Perfectly
67
Q

If a chest x-ray (CXR) is taken immediately after a suspected aspiration event, what is the likely finding?

A) Clear evidence of aspiration pneumonitis
B) No evidence of aspiration pneumonitis
C) Widespread alveolar infiltrates
D) Opacities in the superior segment of the RLL

A

Correct Answer: B) No evidence of aspiration pneumonitis

Rationale: A chest x-ray may not show evidence of aspiration pneumonitis for 6-12 hours following the aspiration event. The radiographic changes take time to develop as the inflammatory response evolves in the lung parenchyma.

How well did you know this?
1
Not at all
2
3
4
5
Perfectly
68
Q

In a patient who has aspirated in the supine position, where is the radiographic evidence of aspiration most likely to be found?

A) The basal segments of the lower lobes
B) The apical segments of the upper lobes
C) The superior segment of the right lower lobe (RLL)
D) The middle lobe and lingula

A

Correct Answer: C) The superior segment of the right lower lobe (RLL)

Rationale: If a patient aspirates in the supine position, the aspirated material is most likely to be found in the superior segment of the right lower lobe due to the anatomical orientation of the bronchial tree and the effect of gravity on the aspirated contents.

How well did you know this?
1
Not at all
2
3
4
5
Perfectly
69
Q

In the event of an aspiration, what is the first step in patient management?

A) Administer a broad-spectrum antibiotic immediately.
B) Suction the oropharynx and turn the patient to the side.
C) Start positive pressure ventilation to clear the aspirate.
D) Perform a bronchoscopy to assess the extent of aspiration.

A

Correct Answer: B) Suction the oropharynx and turn the patient to the side.

Rationale: If aspiration is noted, immediate suctioning of the oropharynx and turning the patient to the side are important to prevent further aspiration and to clear the already aspirated material from the oropharynx.

How well did you know this?
1
Not at all
2
3
4
5
Perfectly
70
Q

What is the role of the Trendelenburg (T-burg) position in managing aspiration?

A) It can stop the reflux of gastric contents.
B) It can facilitate the return of aspirated contents back to the stomach.
C) It prevents aspiration once gastric contents are in the pharynx.
D) It is used to induce vomiting and clear the airway.

A

Correct Answer: C) It prevents aspiration once gastric contents are in the pharynx.

Rationale: The Trendelenburg position will not stop an active reflux, but if gastric contents have refluxed into the pharynx, it can prevent further aspiration by using gravity to keep the contents from moving into the lower respiratory tract.

How well did you know this?
1
Not at all
2
3
4
5
Perfectly
71
Q

Why might gastric fluid pH measurement be useful in chemical pneumonitis?

A) It determines the viscosity of the aspirated fluid.
B) It helps in choosing the appropriate antibiotic.
C) It reflects the pH of the aspirated fluid and its potential for causing lung injury.
D) It indicates the presence of blood in the aspirated material.

A

Correct Answer: C) It reflects the pH of the aspirated fluid and its potential for causing lung injury.

Rationale: Measurement of gastric fluid pH is useful in chemical pneumonitis because the acidity of the aspirated fluid can cause lung injury. A lower pH is associated with a higher risk of chemical injury to the lungs.

How well did you know this?
1
Not at all
2
3
4
5
Perfectly
72
Q

Why is lavage typically not useful in the treatment of aspiration pneumonitis?

A) The aspirated material is too viscous to be removed by lavage.
B) Aspirated gastric fluid is rapidly redistributed in the lungs, making it difficult to remove.
C) Lavage can exacerbate the chemical injury to the lung.
D) Lavage increases the risk of spreading infection.

A

Correct Answer: B) Aspirated gastric fluid is rapidly redistributed in the lungs, making it difficult to remove.

Rationale: Once gastric fluid is aspirated, it quickly spreads to peripheral lung regions, making it difficult to remove by lavage. Additionally, lavage may not be effective in removing the fluid and could potentially exacerbate the situation by spreading it further or causing additional trauma.

How well did you know this?
1
Not at all
2
3
4
5
Perfectly
73
Q

When might antibiotics be considered in the management of aspiration pneumonitis?

A) As an immediate preventive measure following aspiration
B) If the patient remains symptomatic after 48 hours and has positive culture results
C) If gastric fluid pH is very low
D) They are always indicated regardless of symptoms and culture results

A

Correct Answer: B) If the patient remains symptomatic after 48 hours and has positive culture results

Rationale: Antibiotics are not routinely used in the initial treatment of aspiration pneumonitis because there is no evidence that they decrease the incidence of pulmonary infection or alter outcomes. They may be considered if a patient remains symptomatic after 48 hours and has positive culture results, indicating a secondary bacterial infection.

How well did you know this?
1
Not at all
2
3
4
5
Perfectly
74
Q

E-cigarette Vaping Associated Lung Injury (EVALI) is commonly associated with the inhalation of which substance?

A) Alcohol
B) Tetrahydrocannabinol (THC)
C) Caffeine
D) Salbutamol

A

Correct Answer: B) Tetrahydrocannabinol (THC)

Rationale: EVALI has been associated with additives in vaping products, most notably tetrahydrocannabinol (THC), vitamin E acetate, and other oils. These substances have been implicated in causing lung injury among e-cigarette users.

How well did you know this?
1
Not at all
2
3
4
5
Perfectly
75
Q

Which group of symptoms is commonly observed in patients with EVALI?

A) Joint pain, rash, and weight loss
B) Dyspnea, cough, and chest pain
C) Hypertension, bradycardia, and hyperglycemia
D) Diarrhea, jaundice, and hallucinations

A

Correct Answer: B) Dyspnea, cough, and chest pain

Rationale: The typical symptoms of EVALI include respiratory symptoms such as dyspnea, cough, and chest pain. Patients may also experience systemic symptoms like nausea, vomiting, diarrhea, abdominal pain, and fever. Pt may be febrile, tachycardia, tachypnea, and hypoxic

How well did you know this?
1
Not at all
2
3
4
5
Perfectly
76
Q

The radiologic findings in EVALI are similar to those seen in which other pulmonary condition?

A) Pulmonary embolism
B) Pneumonia
C) Acute Respiratory Distress Syndrome (ARDS)
D) Chronic Obstructive Pulmonary Disease (COPD)

A

Correct Answer: C) Acute Respiratory Distress Syndrome (ARDS)

Rationale: Radiologic findings in EVALI often resemble the diffuse alveolar damage seen in ARDS, which includes bilateral opacities on imaging studies without evidence of cardiac failure or fluid overload.

How well did you know this?
1
Not at all
2
3
4
5
Perfectly
77
Q

What is the mainstay of treatment for EVALI?

A) Antifungal medication and chest physiotherapy
B) Antibiotics, systemic steroids, and supportive care
C) Antiviral therapy and supplemental oxygen
D) Immediate cessation of vaping and bronchodilators

A

Correct Answer: B) Antibiotics, systemic steroids, and supportive care

Rationale: The treatment for EVALI typically includes the administration of antibiotics to cover potential secondary bacterial infection, systemic steroids to reduce inflammation, and supportive care, which may include oxygen therapy, mechanical ventilation, and management of associated symptoms.

How well did you know this?
1
Not at all
2
3
4
5
Perfectly
78
Q

What clinical presentation might warrant the consideration of antibiotics in the management of EVALI?

A) The patient has a history of vaping THC-containing products.
B) The patient demonstrates systemic symptoms such as fever or tachycardia.
C) Radiographic findings show clear signs of lung injury.
D) The patient does not respond to standard bronchodilator treatment.

A

Correct Answer: B) The patient demonstrates systemic symptoms such as fever or tachycardia.

Rationale: While there is no direct evidence that antibiotics decrease the incidence of pulmonary infection or alter outcomes in EVALI, they may be considered if the patient presents with systemic symptoms such as fever or tachycardia, which could indicate a possible bacterial superinfection.

How well did you know this?
1
Not at all
2
3
4
5
Perfectly
79
Q

What long-term pulmonary sequelae can survivors of severe COVID-19 experience?

A) Transient bronchospasm and increased mucous production
B) Persistent inflammatory interstitial lung disease
C) Immediate recovery of lung function post-infection
D) Increased susceptibility to bacterial pneumonia only

A

Correct Answer: B) Persistent inflammatory interstitial lung disease

Rationale: Survivors of severe COVID-19 can experience long-term pulmonary complications, including persistent inflammatory interstitial lung disease, which can manifest as fibrosis and continued respiratory symptoms such as dyspnea.

How well did you know this?
1
Not at all
2
3
4
5
Perfectly
80
Q

Which pulmonary function test finding is most commonly reported among COVID-19 survivors with lung involvement?

A) Increased lung compliance
B) Normal diffusion capacity
C) A drop in diffusion capacity
D) Decreased peak expiratory flow rates

A

Correct Answer: C) A drop in diffusion capacity

Rationale: The most commonly reported pulmonary function test finding among COVID-19 survivors is a drop in diffusion capacity, which reflects the severity of initial disease and possible damage to the alveolar-capillary membrane.

How well did you know this?
1
Not at all
2
3
4
5
Perfectly
81
Q

Patients who have required what level of care during acute COVID-19 are at the highest risk for long-term pulmonary complications?

A) Outpatient management with oral medications
B) Mechanical ventilation
C) Brief hospitalization without oxygen therapy
D) Inhaled corticosteroid therapy

A

Correct Answer: B) Mechanical ventilation

Rationale: Patients who have needed mechanical ventilation during their acute COVID-19 illness are at the highest risk for developing long-term pulmonary complications, due to the severity of their respiratory involvement.

How well did you know this?
1
Not at all
2
3
4
5
Perfectly
82
Q

What are common findings and imaging studies such as CT scans in survivors of severe COVID-19?

A) Clear lungs with no residual changes but decreased exercise capacity
B) Decreased exercise capacity, hypoxia, and opacities
C) Enlarged cardiac silhouette and pleural effusions w/ rapid shallow breathing
D) Bronchial wall thickening and tree-in-bud patterns and increased PVR

A

Correct Answer: B) Decreased exercise capacity, hypoxia, and opacities

Rationale: CT scans of COVID-19 survivors often show residual opacities indicative of inflammation or fibrosis, along with clinical findings of decreased exercise capacity and hypoxia, which align with restrictive lung disease.

How well did you know this?
1
Not at all
2
3
4
5
Perfectly
83
Q

What is a direct correlation found in COVID-19 survivors concerning the initial disease process?

A) The severity of initial symptoms correlates with the speed of recovery.
B) There is no correlation between initial disease severity and long-term outcomes.
C) The severity of the initial disease process correlates with a drop in diffusion capacity.
D) The worse the disease, the greater the risk of developing extra-parenchymal lung disease.

A

Correct Answer: C) The severity of the initial disease process correlates with a drop in diffusion capacity.

Rationale: There is a direct relationship between the severity of the initial COVID-19 disease process and subsequent pulmonary function, where a more severe initial presentation correlates with a greater drop in diffusion capacity, indicative of long-term lung damage.

How well did you know this?
1
Not at all
2
3
4
5
Perfectly
84
Q

Acute Respiratory Failure (ARF) is diagnosed when PaO2 is below what threshold?

A) 80 mmHg
B) 70 mmHg
C) 60 mmHg
D) 50 mmHg

A

Correct Answer: C) 60 mmHg

Rationale: ARF is present when the partial pressure of oxygen (PaO2) is less than 60 mmHg despite oxygen supplementation and in the absence of a right-to-left intracardiac shunt.

How well did you know this?
1
Not at all
2
3
4
5
Perfectly
85
Q

A PaCO2 greater than what value, in the absence of respiratory-compensated metabolic alkalosis, is consistent with ARF?

A) 40 mmHg
B) 45 mmHg
C) 50 mmHg
D) 55 mmHg

A

Correct Answer: C) 50 mmHg

Rationale: A PaCO2 greater than 50 mmHg in the absence of respiratory-compensated metabolic alkalosis is consistent with the diagnosis of ARF, indicating hypoventilation and the inability to remove carbon dioxide effectively.

How well did you know this?
1
Not at all
2
3
4
5
Perfectly
86
Q

What characterizes acute respiratory failure (ARF) in terms of blood gases?

A) Decreased PaCO2 and increased pH
B) Unchanged PaCO2 and decreased pH
C) Increased PaCO2 and decreased pH
D) Increased PaCO2 and increased pH

A

Correct Answer: C) Increased PaCO2 and decreased pH

Rationale: ARF is characterized by an abrupt increase in PaCO2 and a decrease in pH, reflecting respiratory acidosis due to acute impairment of ventilation.

How well did you know this?
1
Not at all
2
3
4
5
Perfectly
87
Q

In chronic respiratory failure, the PaCO2 and pH levels demonstrate what relationship?

A) PaCO2 decreased, pH increased
B) PaCO2 increased, pH decreased
C) PaCO2 increased, pH normal
D) PaCO2 normal, pH increased

A

Correct Answer: C) PaCO2 increased, pH normal

Rationale: In chronic respiratory failure, the PaCO2 is increased, but the pH is normal. This indicates a chronic process where the kidneys have had time to compensate for the respiratory acidosis by increasing bicarbonate, thereby normalizing the pH.

How well did you know this?
1
Not at all
2
3
4
5
Perfectly
88
Q

What are the three treatment goals for acute respiratory failure?

A) Airway stabilization, CO2 retention, and pH normalization
B) Patent airway, correction of hypoxemia, and removal of excess CO2
C) Bronchodilation, fluid administration, and forced diuresis
D) Oxygen administration, nutritional support, and sedation

A

Correct Answer: B) Patent airway, correction of hypoxemia, and removal of excess CO2

Rationale: The three primary treatment goals for ARF are to ensure a patent airway, correct hypoxemia, and remove excess CO2. These goals are directed at stabilizing the patient’s respiratory status and improving gas exchange.

How well did you know this?
1
Not at all
2
3
4
5
Perfectly
89
Q

Oxygen therapy can be delivered via various devices. Which of the following typically provides oxygen concentrations greater than 50%?

A) Nasal cannula (NC)
B) Venturi mask
C) Nonrebreather mask
D) T-piece

A

These devices seldom provide 02 concentrations >50%, therefore are only helpful in mild to moderate V/Q mismatching

what the slide says but I think C. is actually 60-100% concentration.

How well did you know this?
1
Not at all
2
3
4
5
Perfectly
90
Q

Continuous Positive Airway Pressure (CPAP) therapy may be initiated when which criteria are not met?

A) PaO2 exceeds 70 mmHg
B) PaO2 falls below 60 mmHg
C) SpO2 falls below 85%
D) Respiratory rate exceeds 24 breaths per minute

A

Correct Answer: B) PaO2 falls below 60 mmHg

Rationale: CPAP may be initiated when methods such as oxygen delivery via nasal cannula or masks fail to maintain a PaO2 above 60 mmHg. CPAP helps by increasing lung volumes and reducing shunting.

How well did you know this?
1
Not at all
2
3
4
5
Perfectly
91
Q

CPAP has what primary effect on lung physiology?

A) It decreases lung volumes by promoting alveolar collapse.
B) It increases lung volumes by opening collapsed alveoli.
C) It increases right-to-left intrapulmonary shunting.
D) It decreases respiratory drive by hyperoxygenation.

A

Correct Answer: B) It increases lung volumes by opening collapsed alveoli.

Rationale: CPAP helps to increase lung volumes by delivering continuous positive pressure to the airways, which can help to open collapsed alveoli and improve oxygenation in conditions with V/Q mismatch.

How well did you know this?
1
Not at all
2
3
4
5
Perfectly
92
Q

What risk is associated with the use of CPAP via face mask?

A) A reduced risk of hypoxemia
B) An increased risk of aspiration, particularly in patients with nausea and vomiting
C) Decreased intrathoracic pressure
D) An increased risk of pneumothorax

A

Correct Answer: B) An increased risk of aspiration, particularly in patients with nausea and vomiting

Rationale: CPAP via face mask can increase the risk of aspiration, especially in patients experiencing nausea and vomiting, due to the positive pressure that may force gastric contents into the pharynx and then into the lungs.

How well did you know this?
1
Not at all
2
3
4
5
Perfectly
93
Q

Maintaining a PaO2 >60 mmHg is considered adequate because it corresponds to an SpO2 of what percentage?

A) Greater than 95%
B) Greater than 90%
C) 85-90%
D) 80-85%

A

Correct Answer: B) Greater than 90%

Rationale: Maintaining a PaO2 greater than 60 mmHg is generally considered adequate because it typically corresponds to an oxygen saturation (SpO2) greater than 90%, which ensures sufficient oxygen delivery to the tissues.

How well did you know this?
1
Not at all
2
3
4
5
Perfectly
94
Q

In volume-cycled ventilation, what is the primary control variable?

A) Respiratory rate (RR)
B) Oxygen concentration (FiO2)
C) Tidal volume (TV)
D) Positive end-expiratory pressure (PEEP)

A

Correct Answer: C) Tidal volume (TV)

Rationale: In volume-cycled ventilation, the primary control variable is the tidal volume (TV), which is set to deliver a fixed volume of air with each mechanical breath, regardless of the pressure (dependent variable) needed to deliver that volume.

How well did you know this?
1
Not at all
2
3
4
5
Perfectly
95
Q

What does a pressure limit in a volume-cycled ventilation system do?

A) Increases the tidal volume delivered to the patient
B) Decreases the respiratory rate to prevent hyperventilation
C) Prevents excessive airway pressure by stopping the flow of gas
D) Reduces the oxygen concentration in the ventilator circuit

A

Correct Answer: C) Prevents excessive airway pressure by stopping the flow of gas

Rationale: A pressure limit can be set in volume-cycled ventilation to prevent excessive inflation pressure. When this set limit is reached, the pressure relief valve opens to stop further gas flow, protecting the patient from high airway pressures.

How well did you know this?
1
Not at all
2
3
4
5
Perfectly
96
Q

Significant increases in Pulmonary Artery Pressure (PAP) during mechanical ventilation may indicate what conditions?

A) Improvement in lung compliance
B) Successful recruitment of alveoli
C) Worsening pulmonary edema, pneumothorax, or obstruction in the endotracheal tube
D) Proper positioning of the endotracheal tube

A

Correct Answer: C) Worsening pulmonary edema, pneumothorax, or obstruction in the endotracheal tube

Rationale: Significant increases in PAP can be a sign of worsening pulmonary edema, the presence of a pneumothorax, a kink in the endotracheal tube (ETT), or a mucus plug obstructing the airway. These conditions can lead to increased resistance to ventilation and increased PAP.

How well did you know this?
1
Not at all
2
3
4
5
Perfectly
97
Q

What is a disadvantage of volume-cycled ventilation?

A) It does not allow for spontaneous breathing.
B) It provides inconsistent tidal volumes.
C) It cannot compensate for leaks in the delivery system.
D) It always requires a high level of sedation.

A

Correct Answer: C) It cannot compensate for leaks in the delivery system.

Rationale: A disadvantage of volume-cycled ventilation is the inability to compensate for leaks in the ventilation system. If a leak occurs, the set tidal volume may not be completely delivered to the patient’s lungs, potentially compromising ventilation.

How well did you know this?
1
Not at all
2
3
4
5
Perfectly
98
Q

What are the primary modes of volume-cycled ventilation?

A) Continuous Mandatory Ventilation (CMV) and Pressure Support Ventilation (PSV)
B) Assisted/Controlled (A/C) ventilation and Synchronized Intermittent Mandatory Ventilation (SIMV)
C) High-Frequency Oscillatory Ventilation (HFOV) and Bilevel Positive Airway Pressure (BiPAP)
D) Pressure Controlled Ventilation (PCV) and Airway Pressure Release Ventilation (APRV)

A

Correct Answer: B) Assisted/Controlled (A/C) ventilation and Synchronized Intermittent Mandatory Ventilation (SIMV)

Rationale: The primary modes of volume-cycled ventilation are Assisted/Controlled (A/C) ventilation, where the ventilator fully supports each breath, and Synchronized Intermittent Mandatory Ventilation (SIMV), which allows for spontaneous breathing efforts between the mandatory ventilator-delivered breaths.

How well did you know this?
1
Not at all
2
3
4
5
Perfectly
99
Q

In Assisted/Controlled (A/C) ventilation, what triggers the ventilator to deliver a breath if there is no inspiratory effort by the patient?

A) A drop in oxygen saturation
B) The passage of a set time interval
C) A rise in carbon dioxide levels
D) The patient’s heart rate

A

Correct Answer: B) The passage of a set time interval

Rationale: In A/C ventilation, if the patient does not initiate an inspiratory effort, the ventilator will automatically deliver a breath at a preset rate based on the passage of a set time interval.

How well did you know this?
1
Not at all
2
3
4
5
Perfectly
100
Q

What is the primary feature of Synchronized Intermittent Mandatory Ventilation (SIMV)?

A) It allows for spontaneous breathing only at a set respiratory rate.
B) It provides mandatory breaths in synchrony with the patient’s inspiratory efforts.
C) It completely takes over the breathing process without allowing for any patient effort.
D) It synchronizes ventilation with the patient’s heart rate.

A

Correct Answer: B) It provides mandatory breaths in synchrony with the patient’s inspiratory efforts.

Rationale: SIMV permits spontaneous ventilation (SV) while providing a predefined minute ventilation. The ventilator circuit provides sufficient gas flow and delivers periodic mandatory breaths that are synchronized with the patient’s spontaneous inspiratory efforts.

How well did you know this?
1
Not at all
2
3
4
5
Perfectly
101
Q

What are the theoretical advantages of SIMV over A/C ventilation?

A) It increases the work of breathing and prevents respiratory muscle atrophy.
B) It allows for complete control of the patient’s ventilation by the provider.
C) It encourages the use of respiratory muscles, potentially improving patient-ventilator coordination.
D) It provides a higher mean airway pressure for better oxygenation.

A

Correct Answer: C) It encourages the use of respiratory muscles, potentially improving patient-ventilator coordination.

Rationale: The theoretical advantages of SIMV over A/C include the continued use of respiratory muscles, which may prevent muscle atrophy, lower mean airway and intrathoracic pressure, prevention of respiratory alkalosis, and potentially improved patient-ventilator coordination.

How well did you know this?
1
Not at all
2
3
4
5
Perfectly
102
Q

How does pressure-cycled ventilation differ from volume-cycled ventilation?

A) It delivers a fixed tidal volume regardless of the pressure required.
B) It provides gas flow until a preset airway pressure is reached.
C) It is primarily used during the weaning process from mechanical ventilation.
D) It prevents spontaneous breathing efforts by the patient.

A

Correct Answer: B) It provides gas flow until a preset airway pressure is reached.

Rationale: Pressure-cycled ventilation differs from volume-cycled by delivering gas flow to the lungs until a preset airway pressure is reached. The tidal volume (dependent variable) varies with changes in compliance and airway resistance, and is not fixed as in volume-cycled ventilation.

How well did you know this?
1
Not at all
2
3
4
5
Perfectly
103
Q

What is the most important predisposing factor for developing ventilator-associated pneumonia (VAP) in mechanically ventilated patients?

A) Patient immunosuppression
B) Mechanical ventilation itself
C) Intubation
D) The type of mechanical ventilator used

A

Correct Answer: C) Intubation

Rationale: In mechanically ventilated patients, intubation is the most important predisposing factor for developing nosocomial pneumonia, also known as ventilator-associated pneumonia (VAP). The process of intubation can allow microorganisms to bypass normal upper airway defenses, leading to infection.

How well did you know this?
1
Not at all
2
3
4
5
Perfectly
104
Q

The primary cause of ventilator-associated pneumonia (VAP) is attributed to what mechanism?

A) Hematogenous spread of bacteria
B) Airborne transmission to the patient
C) Micro-aspiration of contaminated secretions around the endotracheal tube cuff
D) Direct inoculation during surgical procedures

A

Correct Answer: C) Micro-aspiration of contaminated secretions around the endotracheal tube cuff

Rationale: VAP is primarily caused by the micro-aspiration of contaminated secretions around the endotracheal tube (ETT) cuff. These secretions can harbor pathogens that may then enter the lower respiratory tract and cause infection.

How well did you know this?
1
Not at all
2
3
4
5
Perfectly
105
Q

How is nosocomial sinusitis best treated in mechanically ventilated patients?

A) Immediate removal of the endotracheal tube
B) Use of systemic decongestants and head elevation
C) Increased frequency of suctioning
D) Administering prophylactic antiviral medications

A

Correct Answer: B) Use of systemic decongestants and head elevation

Rationale: The treatment of nosocomial sinusitis in mechanically ventilated patients may include antibiotics, systemic decongestants to reduce mucosal swelling, replacement of nasal tubes with oral tubes to prevent obstruction of sinus drainage, and head elevation to facilitate sinus drainage.

106
Q

What conditions are indicative of barotrauma in mechanically ventilated patients?

A) Spontaneous bacterial peritonitis
B) Subcutaneous emphysema and pneumomediastinum
C) Refractory hypoxemia without radiographic changes
D) Increased peak inspiratory pressures without other symptoms

A

Correct Answer: B) Subcutaneous emphysema and pneumomediastinum

Rationale: Barotrauma from mechanical ventilation may present with subcutaneous emphysema, pneumomediastinum, pneumoperitoneum, pneumopericardium, pulmonary interstitial emphysema, arterial gas embolism, or tension pneumothorax. These conditions are due to the passage of air from ruptured alveoli into the extra-alveolar spaces.

107
Q

How does infection increase the risk of barotrauma in mechanically ventilated patients?

A) By increasing the patient’s metabolic demands and oxygen consumption
B) By causing allergic reactions and airway constriction
C) By weakening the pulmonary tissue and making it more susceptible to injury
D) By inducing bronchoconstriction and increasing airway resistance

A

Correct Answer: C) By weakening the pulmonary tissue and making it more susceptible to injury

Rationale: Infections can weaken the structural integrity of the pulmonary tissue, making it more susceptible to rupture and subsequent barotrauma when exposed to the positive pressures of mechanical ventilation.

108
Q

What is a common cause of hypoxemia during mechanical ventilation that is not responsive to an increase in FiO2?

A) Bronchospasm
B) Atelectasis
C) Pulmonary embolism
D) Cardiac shunting

A

Correct Answer: B) Atelectasis

Rationale: Atelectasis, or the collapse of lung tissue, is a common cause of hypoxemia during mechanical ventilation. When atelectasis is present, increasing the fraction of inspired oxygen (FiO2) may not effectively improve oxygenation because the area of the lung affected is not participating in gas exchange. (this is bullshit, unless every single alveoli collapse)

109
Q

In the case of acute desaturation in a mechanically ventilated patient, what should be checked first?

A) Oxygen tank levels
B) Endotracheal tube (ETT) for migration, kinks, or mucous plugs
C) Ventilator settings for accidental changes
D) Patient’s hemoglobin levels

A

Correct Answer: B) Endotracheal tube (ETT) for migration, kinks, or mucous plugs

Rationale: In the event of acute desaturation, it is important to check the ETT for migration, kinks, or occlusion by mucous plugs, as these can obstruct airflow and lead to hypoxemia.

110
Q

Besides atelectasis, what other conditions should be considered in the differential diagnosis of sudden hypoxemia in ventilated patients?

A) Tension pneumothorax (PTX) and pulmonary embolism (PE)
B) Left ventricular failure and myocardial infarction
C) Acute respiratory distress syndrome (ARDS) and sepsis
D) Pleural effusion and pulmonary contusion

A

Correct Answer: A) Tension pneumothorax (PTX) and pulmonary embolism (PE)

Rationale: Sudden hypoxemia in mechanically ventilated patients may also be caused by conditions such as tension pneumothorax and pulmonary embolism, which are often accompanied by hypotension (HoTN) and require immediate attention.

111
Q

What imaging technique can identify atelectasis at the bedside in mechanically ventilated patients?

A) Chest x-ray (CXR)
B) Computed tomography (CT) scan
C) Bedside lung ultrasound (LUS)
D) Magnetic resonance imaging (MRI)

A

Correct Answer: C) Bedside lung ultrasound (LUS)

Rationale: Bedside lung ultrasound (LUS) can be used to identify atelectasis in mechanically ventilated patients. It is a non-invasive tool that can show static air bronchograms indicative of atelectasis.

112
Q

What is the purpose of bronchoscopy in mechanically ventilated patients with suspected atelectasis?

A) To provide oxygenation directly to the alveoli
B) To dilate the bronchi and improve airflow
C) To remove mucous plugs that may be obstructing the airways
D) To decrease inflammation and treat underlying asthma

A

Correct Answer: C) To remove mucous plugs that may be obstructing the airways

Rationale: Bronchoscopy may be necessary to remove mucous plugs in mechanically ventilated patients. Mucous plugs can obstruct the airways and contribute to atelectasis, and their removal is crucial for restoring ventilation to the affected lung areas.

113
Q

What does the arterial partial pressure of oxygen (PaO2) primarily reflect in the context of mechanical ventilation?

A) The ventilatory rate set on the machine
B) The level of peep applied
C) The adequacy of oxygen exchange across alveolar capillaries
D) The amount of oxygen therapy being administered

A

Correct Answer: C) The adequacy of oxygen exchange across alveolar capillaries

Rationale: PaO2 is a measure of the oxygen level in the blood and reflects how well oxygen is able to move from the lungs to the blood, indicating the efficacy of the alveolar capillary oxygen exchange process.

114
Q

What is the significance of the alveolar-arterial (A-a) oxygen gradient?

A) It indicates the effectiveness of the ventilator settings.
B) It measures the difference between alveolar PaO2 and arterial PaO2, helping to assess gas exchange.
C) It determines the concentration of oxygen in the inspired air.
D) It assesses the patient’s metabolic rate and oxygen consumption.

A

Correct Answer: B) It measures the difference between alveolar PaO2 and arterial PaO2, helping to assess gas exchange.

Rationale: The A-a oxygen gradient helps in assessing the efficacy of gas exchange within the lungs by measuring the difference between the alveolar concentration of oxygen (PaO2) and the arterial concentration of oxygen (PaO2). An increased A-a gradient can indicate issues with gas exchange due to various pulmonary pathologies.

115
Q

What response in arterial hypoxemia mechanisms indicates an improvement upon supplemental oxygen administration?

A) A poor response in the case of a right-to-left shunt
B) No change in the case of hypoventilation
C) Improvement in all mechanisms
D) A decrease in PaCO2 levels across all mechanisms

A

Correct Answer: C) Improvement in all mechanisms

Rationale: When supplemental oxygen is provided, there is typically an improvement in arterial hypoxemia across various mechanisms such as hypoventilation, V/Q mismatch, or diffusion impairment. However, the degree of improvement can vary, with conditions like right-to-left shunts showing a poor response compared to other causes.

116
Q

At what PaO2 level does significant desaturation of arterial blood typically occur?

A) >80 mmHg
B) <70 mmHg
C) <60 mmHg
D) <50 mmHg

A

Correct Answer: C) <60 mmHg

Rationale: Significant desaturation of arterial blood typically occurs when the PaO2 falls below 60 mmHg, indicating a critical level of hypoxemia that stimulate compensatory responses.

At a partial pressure of oxygen (PaO2) of 60 mmHg, arterial blood is typically around 90% saturated with oxygen

117
Q

Which condition is NOT generally improved by increasing the fraction of inspired oxygen (FiO2)?

A) Ventilation/perfusion (V/Q) mismatch
B) Right-to-left pulmonary shunting
C) Hypoventilation
D) Diffusion limitation due to interstitial lung disease

A

Correct Answer: B) Right-to-left pulmonary shunting

Rationale: While increasing the FiO2 can improve the PaO2 in conditions like V/Q mismatch and hypoventilation, it has minimal to no effect in the case of significant right-to-left pulmonary shunting, where blood bypasses the oxygenation process in the lungs.

118
Q

What triggers compensatory responses to acute hypoxemia?

A) An increase in PaCO2
B) A decrease in PaO2 to less than 60 mmHg
C) A stable PaO2 above 80 mmHg
D) An increase in pH level

A

Correct Answer: B) A decrease in PaO2 to less than 60 mmHg

Rationale: Compensatory responses are stimulated by an acute decrease in PaO2 below 60 mmHg, as the body attempts to maintain adequate tissue oxygenation despite the low arterial oxygen levels.

119
Q

In chronic hypoxemia, when are compensatory responses activated?

A) When PaO2 is below 60 mmHg
B) When PaO2 is below 50 mmHg
C) When PaO2 is above 70 mmHg
D) When PaO2 is stable regardless of actual value

A

Correct Answer: B) When PaO2 is below 50 mmHg

Rationale: In chronic hypoxemia, compensatory responses such as carotid body-induced hyperventilation, hypoxic pulmonary vasoconstriction, and increased sympathetic nervous system activity are typically activated when PaO2 falls below 50 mmHg. These responses help to maintain oxygen delivery to the tissues.

120
Q

What are the physiological responses to chronic hypoxemia?

A) Decreased respiratory drive and vasodilation
B) Increased cardiac output (COP) and alveolar ventilation
C) Pulmonary hypertension and erythropoiesis
D) Metabolic alkalosis and respiratory acidosis

A

Correct Answer: B) Increased cardiac output (COP) and alveolar ventilation

Rationale: Chronic hypoxemia leads to compensatory mechanisms to increase oxygen delivery, including increased alveolar ventilation (stimulated by the carotid body), hypoxic pulmonary vasoconstriction to divert blood away from hypoxic areas of the lung, and increased sympathetic nervous system activity to enhance cardiac output and peripheral oxygen delivery.

121
Q

Which of the following conditions is likely to result in a physiological increase in dead space ventilation (VD:VT ratio)?
A) Pneumonectomy
B) Pneumonia
C) Pulmonary embolism
D) Increased physical fitness

A

Correct Answer: C) Pulmonary embolism

Rationale: Pulmonary embolism is a condition that can create areas in the lungs with adequate ventilation but inadequate perfusion, thereby increasing the dead space ventilation. This is because the blood clots block the pulmonary vasculature, leading to areas where alveoli are ventilated but not perfused, thus not participating in gas exchange, which is the definition of dead space.

122
Q

A patient with chronic obstructive pulmonary disease (COPD) is most likely to exhibit which alteration in VD:VT ratio?
A) A decrease in VD:VT ratio due to hyperventilation.
B) No change in VD:VT ratio as COPD does not affect alveolar ventilation.
C) An increase in VD:VT ratio due to air trapping and ventilation-perfusion mismatch.
D) A decrease in VD:VT ratio due to increased RBC mass.

A

Correct Answer: C) An increase in VD:VT ratio due to air trapping and ventilation-perfusion mismatch.

Rationale: COPD often leads to air trapping and a ventilation-perfusion mismatch due to obstruction of airways and destruction of lung parenchyma. These changes can increase the proportion of ventilation that is not involved in gas exchange, thereby increasing the VD:VT ratio.

123
Q

In the presence of acute respiratory failure (ARF), the VD:VT ratio often increases. Which of the following mechanisms contributes least to this increase?
A) Alveolar capillary damage leading to impaired CO2 transfer
B) Increased erythropoiesis and RBC mass in response to hypoxemia
C) Ventilation-perfusion mismatch due to atelectasis or pneumonia
D) Reduction in cardiac output leading to decreased perfusion

A

Correct Answer: B) Increased erythropoiesis and RBC mass in response to hypoxemia

Rationale: While increased RBC mass is a compensatory mechanism in response to chronic hypoxemia to improve oxygen carrying capacity, it does not directly affect the VD:VT ratio. The other options directly relate to ventilation-perfusion mismatch or decreased perfusion, both of which can increase dead space ventilation.

124
Q

What arterial blood gas (ABG) parameter is most reflective of alveolar ventilation?
A) PaO2
B) SaO2
C) PaCO2
D) HCO3-

A

Correct Answer: C) PaCO2

Rationale: The partial pressure of carbon dioxide (PaCO2) in arterial blood gases is directly related to alveolar ventilation. A decrease in alveolar ventilation leads to an increase in PaCO2, whereas an increase in alveolar ventilation will decrease PaCO2. PaO2 and SaO2 reflect the oxygenation status of the blood and HCO3- is more reflective of metabolic processes and compensation for respiratory changes.

125
Q

A patient presenting with dyspnea is found to have a VD:VT ratio of 0.55. Considering the diagnostic threshold mentioned, this value suggests which of the following?

A) Normal respiratory function
B) An efficient compensatory mechanism in chronic hypoxemia
C) A possible increase in physiological dead space indicative of a pulmonary pathology
D) Adequate perfusion and a low likelihood of pulmonary embolism

A

Correct Answer: C) A possible increase in physiological dead space indicative of a pulmonary pathology

Rationale: A VD:VT ratio greater than 0.3 and approaching 0.6 indicates a possible increase in physiological dead space, often associated with pulmonary pathologies such as pulmonary embolism, acute respiratory failure, or a decrease in cardiac output. A normal VD:VT ratio is generally ≤0.3 in healthy individuals. Thus, a value of 0.55 would be considered diagnostic of increased dead space ventilation and suggest a need for further evaluation for underlying pulmonary issues. slide says may increase above >= 0.6

126
Q

Which of the following is an intended effect of permissive hypercapnia?
A) To decrease intracranial pressure (ICP)
B) To reduce cerebral blood flow (CBF)
C) To mitigate respiratory acidosis
D) To avoid potential barotrauma from mechanical ventilation

A

Correct Answer: D) To avoid potential barotrauma from mechanical ventilation

Rationale: Permissive hypercapnia is a strategy that allows a higher than normal PaCO2 to avoid the potentially harmful effects of aggressive mechanical ventilation, such as barotrauma or volutrauma, particularly in patients with conditions like acute respiratory distress syndrome (ARDS). This strategy acknowledges that a certain level of hypercapnia can be tolerated to minimize lung injury from high ventilation pressures or volumes.

127
Q

What complication is most directly associated with acute increases in PaCO2?
A) Metabolic acidosis
B) Decreased cerebral blood flow (CBF)
C) Increased intracranial pressure (ICP)
D) Decreased oxygen saturation (SaO2)

A

Correct Answer: C) Increased intracranial pressure (ICP)

Rationale: Acute increases in PaCO2 can lead to vasodilation of cerebral vessels, which in turn can increase cerebral blood flow (CBF) and consequently intracranial pressure (ICP). This can be particularly detrimental in patients with head injuries or other conditions where increased ICP is dangerous.

128
Q

Hypercarbia is typically defined as a PaCO2 greater than 45mmHg. However, therapeutic strategies sometimes allow for a controlled rise in PaCO2 levels. What is the rationale behind using a higher threshold (e.g., 55mmHg) in the management of certain patients?

A) To hasten the resolution of metabolic alkalosis
B) To reduce the likelihood of acute respiratory distress syndrome (ARDS)
C) To minimize the risk of oxygen toxicity
D) To facilitate protective lung strategies in mechanical ventilation

A

Correct Answer: D) To facilitate protective lung strategies in mechanical ventilation

Rationale: The rationale behind allowing a higher PaCO2 (e.g., 55mmHg) is to enable the use of lower tidal volumes and pressures in mechanical ventilation, which are known as lung protective strategies. These strategies can decrease the risk of ventilator-induced lung injury, including barotrauma and volutrauma, particularly in patients with ARDS.

129
Q

In the context of hypercarbia, symptoms vary depending on both the level and rate of increase in CO2. Which of the following patient presentations would most likely require immediate intervention?

A) A patient with a gradual increase in PaCO2 to 50mmHg over several days exhibiting mild headaches
B) A patient with a rapid increase in PaCO2 to 50mmHg within hours experiencing confusion and lethargy
C) A patient with a stable chronic PaCO2 of 60mmHg with no acute changes in mental status
D) A patient with controlled permissive hypercapnia on mechanical ventilation and stable hemodynamics

A

Correct Answer: B) A patient with a rapid increase in PaCO2 to 50mmHg within hours experiencing confusion and lethargy

Rationale: The rate of increase in PaCO2 is a critical factor in symptom manifestation. A rapid rise in PaCO2 can lead to acute cerebral vasodilation, increased intracranial pressure, and symptoms such as confusion and lethargy, necessitating immediate intervention. In contrast, a gradual increase often allows time for compensatory mechanisms to mitigate symptom severity, and chronic hypercapnia may be well-tolerated if it is stable and the patient is adapted to it.

130
Q

At which level of PaCO2 is a patient most at risk for central nervous system (CNS) depression, necessitating immediate intervention?
A) PaCO2 of 75 mmHg
B) PaCO2 of 65mmHg with permissive hypercapnia
C) PaCO2 of 60mmHg in a chronic obstructive pulmonary disease (COPD) patient
D) PaCO2 of greater than 80mmHg

A

Correct Answer: D) PaCO2 of greater than 80mmHg

Rationale: Extreme increases in PaCO2, such as levels greater than 80mmHg, are associated with significant central nervous system (CNS) depression. This condition can manifest as diminished consciousness or coma and requires immediate intervention to restore normal ventilation and gas exchange to prevent further deterioration of the patient’s neurological status and overall health.

131
Q

What is the primary clinical significance of measuring arterial pH (pHa) in patients?
A) To determine the patient’s blood oxygen carrying capacity
B) To assess for the presence of acidemia or alkalemia and their potential causes
C) To evaluate the efficiency of red blood cell (RBC) production
D) To directly measure arterial carbon dioxide levels

A

Correct Answer: B) To assess for the presence of acidemia or alkalemia and their potential causes

Arterial hypoxemia is assocw/ metabolic acidosis

Rationale: The measurement of arterial pH is essential to detect deviations from the normal pH range, indicating acidemia or alkalemia. Acidemia can be a result of respiratory or metabolic dysfunction, leading to conditions such as dysrhythmias and pulmonary hypertension. Alkalemia can result from respiratory alkalosis due to mechanical hyperventilation and from metabolic causes such as diuretic use leading to chloride and potassium loss. Identifying these pH imbalances is critical for determining the underlying disorder and guiding appropriate treatment.

132
Q

Which of the following is a potential consequence of acidemia on cardiovascular function?
A) Decreased likelihood of dysrhythmias
B) Increased pulmonary vascular resistance
C) Improved cardiac contractility
D) Decreased myocardial oxygen demand

A

Correct Answer: B) Increased pulmonary vascular resistance

Rationale: Acidemia can lead to a number of cardiovascular complications, one of which is increased pulmonary vascular resistance. This is due to the vasoconstrictive effects of acidosis on the pulmonary vasculature, which can also contribute to pulmonary hypertension. These changes can compromise pulmonary circulation and exacerbate respiratory distress.

133
Q

What condition can be exacerbated by alkalemia due to compensatory hypoventilation in patients recovering from acute respiratory failure (ARF)?
A) Metabolic acidosis
B) Pulmonary embolism
C) Ventilator dependence
D) Hypoxemia

A

Correct Answer: C) Ventilator dependence

Rationale: In patients recovering from ARF, alkalemia can occur as a result of compensatory hypoventilation, which is the body’s response to correct the high pH. However, this can pose a challenge in weaning the patient from the ventilator, as the body’s drive to breathe may be reduced due to the correction of alkalosis, leading to prolonged ventilator dependence.

134
Q

A patient undergoing mechanical ventilation is noted to have increased chloride and potassium excretion. This could be an indication of which of the following imbalances?
A) Respiratory acidosis
B) Metabolic acidosis
C) Respiratory alkalosis
D) Metabolic alkalosis

A

Correct Answer: C) Respiratory alkalosis

Rationale: Respiratory alkalosis, which can be caused by mechanical hyperventilation, often results in renal compensation that includes increased excretion of chloride and potassium ions to balance the pH. The loss of these ions can contribute to the development of alkalemia and its associated complications, including dysrhythmias.

135
Q

What percentage of the cardiac output (COP) does a normal physiologic intrapulmonary shunt comprise?
A) 0-1%
B) 2-5%
C) 6-10%
D) 11-15%

A

Correct Answer: B) 2-5%

Rationale: A physiologic shunt is a normal part of pulmonary circulation, typically comprising 2-5% of the cardiac output. It accounts for the small amount of blood that does not participate in gas exchange because it bypasses ventilated alveoli.

136
Q

The calculation of shunt fraction in a patient breathing less than 100% oxygen helps distinguish between:

A) Right-to-left and left-to-right shunting.
B) Intrapulmonary and extrapulmonary shunting.
C) The contributions of V/Q mismatch and intrapulmonary shunting.
D) Metabolic acidosis and respiratory acidosis.

A

Correct Answer: C) The contributions of V/Q mismatch and intrapulmonary shunting.

Rationale: Determining the shunt fraction with a patient breathing less than 100% oxygen allows assessment of both V/Q mismatch and intrapulmonary shunt effects on oxygenation, as both can contribute to hypoxemia.

137
Q

Intrapulmonary shunting is a result of blood passing through which of the following without gas exchange?

A) Adequately ventilated alveoli
B) Nonventilated alveoli
C) Fully oxygenated alveoli
D) Alveoli with high V/Q ratio

A

Correct Answer: B) Nonventilated alveoli

Rationale: Intrapulmonary shunting occurs when blood passes through nonventilated alveoli, leading to hypoxemia due to the mixture of oxygenated and deoxygenated blood.

138
Q

The passage of blood through the bronchial and thebesian veins is an example of which kind of shunt?
A) Physiologic
B) Pathologic
C) Intrapulmonary
D) Extrapulmonary

A

Correct Answer: A) Physiologic

Rationale: Blood that passes directly to the left side of the circulation through the bronchial and thebesian veins is part of the normal physiologic shunting in the lungs.

139
Q

A decrease in PaO2 due to intrapulmonary shunting is primarily due to:
A) Enhanced oxygen delivery to tissues
B) The dilution of oxygenated blood with oxygen-poor blood
C) Reduced hemoglobin affinity for oxygen
D) Hyperventilation of functional alveoli

A

Correct Answer: B) The dilution of oxygenated blood with oxygen-poor blood

Rationale: Intrapulmonary shunting leads to a decrease in PaO2 due to the mixing of oxygenated blood from ventilated alveoli with hypo-oxygenated blood from unventilated alveoli, causing overall dilution.

140
Q

What is the purpose of calculating the shunt fraction while a patient breathes 100% oxygen?

A) To estimate the patient’s response to supplemental oxygen therapy
B) To quantify the degree of V/Q mismatch
C) To isolate the intrapulmonary shunt effect on gas exchange
D) To determine the level of hypoxemia due to hypoventilation

A

Correct Answer: C) To isolate the intrapulmonary shunt effect on gas exchange

Rationale: When a patient breathes 100% oxygen, the contribution of V/Q mismatch to hypoxemia is minimized, allowing for the specific calculation of shunt fraction to reflect only the impact of intrapulmonary shunting. This helps in identifying true shunt physiology, which is important for diagnosing certain pulmonary conditions and guiding appropriate treatment.

141
Q

Which of the following criteria is not part of the guidelines for discontinuing mechanical ventilation?
A) PaO2 of >80 mm Hg with an FiO2 of <0.5
B) Vital capacity of >15 mL/kg
C) Negative inspiratory pressure of more than −20 cmH2O
D) VD:VT of < 0.6

A

Correct Answer: A) PaO2 of >80 mm Hg with an FiO2 of <0.5

Rationale: The correct parameter according to the guidelines for discontinuing mechanical ventilation is a PaO2 of >60 mm Hg with an FiO2 of <0.5, not >80 mm Hg. The other listed criteria are consistent with the guidelines presented for ventilator weaning.

142
Q

A patient being considered for ventilator weaning should demonstrate which respiratory rate (RR) to be deemed ready for a trial of spontaneous ventilation (SV)- per guidelines Max?
A) RR of <15
B) RR of <20
C) RR of <25
D) RR of <30

A

Correct Answer: B) RR of <20

Rationale: The guidelines suggest that a patient should have a respiratory rate (RR) of less than 20 breaths per minute to be considered for a trial of spontaneous ventilation, as part of the criteria for weaning from mechanical ventilation.

143
Q

What does a vital capacity of >15 mL/kg indicate in the context of ventilator weaning?

A) Adequate muscle strength and endurance
B) Compensation for metabolic acidosis
C) Impaired pulmonary function
D) Increased risk of respiratory muscle fatigue

A

Correct Answer: A) Adequate muscle strength and endurance

Rationale: A vital capacity of >15 mL/kg suggests that the patient has sufficient respiratory muscle strength and endurance, which are important indicators of the patient’s ability to breathe independently without mechanical support.

144
Q

In the weaning process, a negative inspiratory pressure (NIP) of more
negative than −20 cmH2O typically indicates:

A) Adequate spontaneous ventilatory drive
B) Ineffective cough reflex
C) Inability to maintain airway patency
D) Risk of airway collapse during inspiration

A

Correct Answer: A) Adequate spontaneous ventilatory drive

Rationale: A negative inspiratory pressure more negative than −20 cmH2O signifies a strong spontaneous ventilatory drive, which is a critical factor in determining a patient’s readiness to wean from mechanical ventilation.

145
Q

Which weaning strategy involves the patient taking over more of the breathing work gradually while still receiving set mandatory breaths from the ventilator?

A) Continuous positive airway pressure (CPAP)
B) Synchronized Intermittent Mandatory Ventilation (SIMV)
C) Pressure support ventilation (PSV)
D) T-piece trials

A

Correct Answer: B) Synchronized Intermittent Mandatory Ventilation (SIMV)

Rationale: SIMV allows the patient to breathe spontaneously while providing a set number of mandatory breaths. The frequency of these mandatory breaths can be progressively reduced as the patient’s spontaneous breathing effort improves, facilitating weaning.

146
Q

A patient demonstrating rapid breathing with low tidal volumes is typically interpreted as:

A) Being ready for extubation
B) Having an adequate spontaneous ventilatory drive
C) Exhibiting signs of potential weaning success
D) Showing signs of respiratory distress and potential weaning failure

A

Correct Answer: D) Showing signs of respiratory distress and potential weaning failure

Rationale: Rapid breathing rates combined with low tidal volumes often indicate inadequate ventilation and the potential for respiratory muscle fatigue, suggesting that the patient may not tolerate extubation well.

147
Q

Intermittent trials of total removal of mechanical support through a T-piece test a patient’s ability to:

A) Maintain airway pressure independently
B) Breathe entirely on their own without assistance
C) Respond to a set number of mandatory ventilator breaths
D) Tolerate gradual decreases in ventilatory support

A

Correct Answer: B) Breathe entirely on their own without assistance

Rationale: The T-piece trial involves the total removal of mechanical support, allowing clinicians to assess the patient’s ability to maintain adequate respiration unassisted, which is a crucial step in determining readiness for extubation.

148
Q

The use of decreasing levels of pressure support ventilation (PSV) aims to:

A) Increase the respiratory rate and reduce the patient’s work of breathing
B) Maintain a constant respiratory rate while adjusting oxygen concentration
C) Gradually reduce the ventilatory assistance as the patient’s respiratory strength improves
D) Provide a set number of mandatory breaths regardless of the patient’s spontaneous breathing

A

Correct Answer: C) Gradually reduce the ventilatory assistance as the patient’s respiratory strength improves - (vent weaning)

Rationale: PSV is used to support the patient’s spontaneous breathing efforts with a preset amount of positive pressure. As the patient’s condition improves, the level of pressure support is gradually reduced, allowing the respiratory muscles to adapt and take on more work.

149
Q

To ensure adequate oxygenation during weaning from mechanical ventilation, the PaO2 should be maintained above what level with a fractional inspired oxygen (FiO2) of less than 0.5?

A) >50 mmHg
B) >60 mmHg
C) >70 mmHg
D) >80 mmHg

A

Correct Answer: B) >60 mmHg

Rationale: The target PaO2 level during weaning is greater than 60 mmHg with an FiO2 of less than 0.5 to ensure sufficient oxygenation without excessive oxygen supplementation.

150
Q

When considering weaning a patient from mechanical ventilation, which of the following is not a parameter to be evaluated?
A) PEEP of <5 cmH2O
B) VC of >15 mL/kg
C) PaCO2 of <60 mmHg
D) RR of <20 breaths/min

A

Correct Answer: C) PaCO2 of <60 mmHg

Rationale: The correct target for PaCO2 when considering ventilator weaning is to remain below 50 mmHg, not 60 mmHg. The other options are established parameters for evaluating a patient’s readiness for weaning.

151
Q

Which of the following patient abilities is essential for considering extubation?
A) Diminished gag reflex
B) Passive secretion clearance
C) Ability to generate an effective cough and clear secretions
D) Dependency on high levels of PEEP

A

Correct Answer: C) Ability to generate an effective cough and clear secretions

Rationale: The ability to generate an effective cough and clear secretions is crucial for a patient’s readiness for extubation. It indicates that the patient may be able to maintain airway patency without mechanical support.

152
Q

Supplemental oxygen is often required after extubation due to:
A) Reduced cardiac output
B) Ventilation-perfusion (V/Q) mismatching
C) Excessive production of CO2
D) Hypercapnia exceeding 60 mmHg

A

Correct Answer: B) Ventilation-perfusion (V/Q) mismatching

Rationale: After extubation, patients often require supplemental oxygen due to V/Q mismatching, which can persist or become evident once the patient is no longer receiving the controlled ventilation support.

153
Q

The weaning process from oxygen supplementation is typically guided by:
A) The patient’s subjective feeling of dyspnea
B) The level of patient consciousness
C) Measurements of PaO2 and/or monitoring of SpO2
D) Patient’s ability to maintain a regular breathing pattern

A

Correct Answer: C) Measurements of PaO2 and/or monitoring of SpO2

Rationale: Oxygen weaning should be guided by objective measurements of arterial oxygenation (PaO2) and the saturation of peripheral oxygen (SpO2), ensuring the patient is safely receiving the appropriate amount of supplemental oxygen.

154
Q

Which condition is most closely associated with the highest risk of developing Acute Respiratory Distress Syndrome (ARDS)?
A) Pneumonia
B) Sepsis
C) Pulmonary contusion
D) Aspiration of gastric contents

A

Correct Answer: B) Sepsis

Rationale: Sepsis is known to be the condition most strongly associated with the highest risk of ARDS due to the systemic inflammatory response and the release of cytokines that can lead to widespread lung damage.

155
Q

What are the characteristic clinical findings of ARDS?
A) Slow-onset respiratory failure and arterial hyperoxemia
B) Rapid-onset respiratory failure and arterial hypoxemia
C) Chronic respiratory failure and normal chest X-ray (CXR) findings
D) Insidious onset of dyspnea and pleural effusion on CXR

A

Correct Answer: B) Rapid-onset respiratory failure and arterial hypoxemia

Rationale: The hallmarks of ARDS include rapid-onset respiratory failure and arterial hypoxemia, with chest X-ray findings that may mimic cardiogenic pulmonary edema despite a non-cardiogenic origin.

156
Q

What is a potential long-term complication of Acute ARDS?

A) Resolution with increased pulmonary compliance
B) Progression to fibrosing alveolitis with persistent arterial hypoxemia
C) Complete recovery with hyperoxemia
D) Chronic bronchitis with reversible airway obstruction

A

Correct Answer: B) Progression to fibrosing alveolitis with persistent arterial hypoxemia

Rationale: While acute ARDS can resolve completely, it may also progress to chronic conditions such as fibrosing alveolitis, characterized by persistent arterial hypoxemia and decreased pulmonary compliance due to lung scarring and fibrosis.

157
Q

Proinflammatory cytokines in ARDS are primarily responsible for:

A) Decreasing alveolar capillary membrane permeability
B) Causing bronchoconstriction and airway narrowing
C) Increasing alveolar capillary membrane permeability and alveolar edema
D) Promoting alveolar repair and regeneration

A

Correct Answer: C) Increasing alveolar capillary membrane permeability and alveolar edema

Rationale: In ARDS, the release of proinflammatory cytokines leads to increased permeability of the alveolar capillary membrane, contributing to alveolar edema and impairing gas exchange.

158
Q

Supportive care in ARDS typically includes all of the following, except:
A) Ventilation with high tidal volumes
B) Antibiotics if infection is suspected or confirmed
C) Stress ulcer prophylaxis
D) Early enteral feeding

A

Correct Answer: A) Ventilation with high tidal volumes

Rationale: Supportive care in ARDS typically avoids high tidal volumes to prevent ventilator-induced lung injury and instead focuses on lung-protective strategies with lower tidal volumes. Antibiotics, stress ulcer prophylaxis, and early enteral feeding are all components of supportive care in ARDS management.

159
Q

What is the primary physiological benefit of prone positioning in the management of ARDS?
A) Reduction of pleural pressure
B) Enhancement of diaphragmatic movement
C) Improvement of V/Q matching by recruitment of lung units
D) Decrease in cardiac output

A

Correct Answer: C) Improvement of V/Q matching by recruitment of lung units

Rationale: Prone positioning is utilized in ARDS to improve oxygenation by recruiting atelectatic lung units and optimizing ventilation-perfusion (V/Q) matching due to the redistribution of pulmonary blood flow and the effects of gravity.

160
Q

Extracorporeal membrane oxygenation (ECMO) is considered for ARDS patients primarily to:

A) Increase tidal volume and respiratory rate
B) Rest the lungs while providing adequate gas exchange
C) Directly reduce proinflammatory cytokines in the bloodstream
D) Stimulate surfactant production in the lungs

A

Correct Answer: B) Rest the lungs while providing adequate gas exchange

Rationale: ECMO is used in the management of severe ARDS to allow the lungs to rest by taking over the gas exchange function, which can be especially beneficial in cases of severe hypoxemia and respiratory acidosis unresponsive to conventional treatment.

161
Q

Which of the following is not typically included in the additional supportive therapies for ARDS?
A) Fluid restriction to manage optimal fluid balance
B) Neuromuscular blockade (NMB) agents
C) Systemic corticosteroids
D) Beta-blocker therapy

A

Correct Answer: D) Beta-blocker therapy

Rationale: While optimal fluid management, the use of neuromuscular blockade agents, and systemic corticosteroids are part of supportive therapies in ARDS, beta-blocker therapy is not typically included in this management strategy.

162
Q

The administration of inhaled nitric oxide in ARDS management is aimed at:
A) Increasing blood oxygen carrying capacity
B) Selectively dilating pulmonary vessels to improve V/Q matching
C) Stimulating respiratory drive
D) Enhancing mucociliary clearance in the airways

A

Correct Answer: B) Selectively dilating pulmonary vessels to improve V/Q matching

Rationale: Inhaled nitric oxide is used in ARDS for its selective pulmonary vasodilation effects, which can improve V/Q matching and oxygenation without causing systemic vasodilation.

163
Q

Recruitment maneuvers in ARDS are used to:
A) Reduce pulmonary vascular resistance
B) Lower intracranial pressure
C) Re-expand collapsed alveoli
D) Increase renal blood flow

A

Correct Answer: C) Re-expand collapsed alveoli

Rationale: Recruitment maneuvers in ARDS involve transient increases in airway pressure to open (or ‘recruit’) collapsed alveoli, thereby improving oxygenation and lung compliance.

164
Q

Which of the following conditions is not typically categorized under interstitial lung diseases (ILD)?
A) Sarcoidosis
B) Chronic bronchitis
C) Hypersensitivity Pneumonitis
D) Pulmonary Langerhans Cell Histiocytosis

A

Correct Answer: B) Chronic bronchitis

Rationale: Chronic bronchitis is characterized by chronic productive cough and is typically classified under obstructive lung diseases, unlike the other conditions listed, which are types of ILD and exhibit restrictive lung physiology.

164
Q

Interstitial lung disease (ILD) is characterized by which type of pulmonary physiology?
A) Obstructive
B) Restrictive
C) Mixed obstructive-restrictive
D) Normal

A

Correct Answer: B) Restrictive

Rationale: ILD is associated with restrictive physiology due to the stiffening of lung parenchyma, which restricts the expansion of the lungs, leading to a decrease in lung volumes.

165
Q

What is a common clinical presentation of patients with chronic intrinsic restrictive lung disease, such as ILD?
A) Wheezing and productive cough
B) Dyspnea and nonproductive cough
C) Sudden onset of chest pain and hypercapnia
D) Hemoptysis and digital clubbing

A

Correct Answer: B) Dyspnea and nonproductive cough

Rationale: Patients with ILD commonly present with dyspnea on exertion and a dry, nonproductive cough due to the underlying parenchymal involvement without significant airway hypersecretion.

166
Q

The development of pulmonary hypertension and cor pulmonale in the setting of ILD is primarily due to:
A) Hyperinflation of alveoli
B) Loss of pulmonary vasculature and fibrosis
C) Increased production of mucus in the bronchi
D) Persistent bronchospasms and air trapping

A

Correct Answer: B) Loss of pulmonary vasculature and fibrosis

Rationale: Progressive pulmonary fibrosis in ILD can cause obliteration of the pulmonary capillary bed, leading to increased pulmonary vascular resistance, which in turn can lead to pulmonary hypertension and cor pulmonale (right-sided heart failure).

167
Q

Sarcoidosis is primarily characterized by the formation of granulomas in various tissues. Which organ system is most commonly affected by this disorder?
A) Gastrointestinal system
B) Musculoskeletal system
C) Upper Respiratory system
D) Lungs and intrathoracic lymph nodes

A

correct answer: D) Lungs and intrathoracic lymph nodes

Rationale: Sarcoidosis most commonly affects the respiratory system, with granulomatous inflammation typically involving the lungs and intrathoracic lymph nodes.

168
Q

Which of the following is a classic cardiac manifestation of sarcoidosis?
A) Atrial fibrillation
B) Myocardial infarction
C) Conduction abnormalities and dysrhythmias
D) Ventricular hypertrophy

A

Correct Answer: C) Conduction abnormalities and dysrhythmias

Rationale: Myocardial sarcoidosis can cause granulomatous changes in the heart, leading to conduction abnormalities and various dysrhythmias due to disruption of the normal electrical pathways within the myocardium.

169
Q

Hypercalcemia in sarcoidosis is due to:
A) Excessive dietary calcium intake
B) Renal failure
C) Increased calcium absorption mediated by vitamin D dysregulation
D) Primary hyperparathyroidism

A

Correct Answer: C) Increased calcium absorption mediated by vitamin D dysregulation

Rationale: In sarcoidosis, hypercalcemia occurs due to the dysregulation of vitamin D by activated macrophages within granulomas, which increases calcium absorption from the gut.

Hypercalcemia is a classic manifestation of sarcoidosis

170
Q

Laryngeal involvement in sarcoidosis is significant because it:
A) Typically leads to chronic hoarseness
B) Is a leading cause of stridor in adult patients
C) Can interfere with intubation
D) Usually results in complete airway obstruction

A

Correct Answer: C) Can interfere with intubation

Rationale: Laryngeal sarcoidosis, which occurs in up to 5% of patients, can cause airway obstruction or distortion due to granulomatous changes, thus potentially complicating intubation efforts.

171
Q

Neurologic sarcoidosis is associated with which of the following clinical features?
A) Bilateral lower extremity weakness
B) Unilateral facial nerve palsy
C) Seizures
D) Intention tremor

A

B) Unilateral facial nerve palsy

172
Q

Which enzyme’s activity is characteristically increased in sarcoidosis due to its production by granuloma cells?
A) Lactate dehydrogenase (LDH)
B) Creatine kinase (CK)
C) Angiotensin-converting enzyme (ACE)
D) Alkaline phosphatase (ALP)

A

Correct Answer: C) Angiotensin-converting enzyme (ACE)

Rationale: Angiotensin-converting enzyme (ACE) activity is often increased in sarcoidosis, as it is produced by the epithelioid cells of granulomas. This increase can be used as a marker in the diagnosis and management of the disease.

173
Q

What is the primary reason corticosteroids are used in the management of sarcoidosis?
A) To stimulate the immune system
B) To reduce inflammation and suppress symptoms
C) To enhance granuloma formation
D) To decrease angiotensin-converting enzyme activity

A

Correct Answer: B) To reduce inflammation and suppress symptoms

Rationale: Corticosteroids are the mainstay treatment for sarcoidosis because of their potent anti-inflammatory effects, which can suppress the granulomatous inflammation that characterizes the disease and treat hypercalcemia often seen in sarcoidosis.

174
Q

A patient with sarcoidosis may require what type of diagnostic procedures to obtain tissue for confirmation of the disease?

A) Echocardiogram and pulmonary function tests
B) Blood cultures and sputum cultures
C) Mediastinoscopy, endobronchial ultrasound, and bronchoscopy
D) Computed tomography (CT) scan and magnetic resonance imaging (MRI)

A

Correct Answer: C) Mediastinoscopy, endobronchial ultrasound, and bronchoscopy

Rationale: Tissue diagnosis of sarcoidosis often requires procedures such as mediastinoscopy, endobronchial ultrasound, and bronchoscopy to obtain tissue samples or lavage fluid for histological examination.

175
Q

Which clinical manifestation is considered a classic but less common feature of sarcoidosis?
A) Hypercalcemia
B) Hypoglycemia
C) Hypocalcemia
D) Hyperglycemia

A

Correct Answer: A) Hypercalcemia

Rationale: Hypercalcemia is a classic manifestation of sarcoidosis, although it is not present in all patients. It results from increased production of calcitriol by activated macrophages in granulomas.

176
Q

The development of advanced pulmonary fibrosis in sarcoidosis may lead to:
A) A reduction in pulmonary arterial pressure
B) Normal to increased V/Q ratio
C) Development of pulmonary hypertension
D) Increase in lung compliance

A

Correct Answer: C) Development of pulmonary hypertension

Rationale: Progressive pulmonary fibrosis can lead to obliteration of the pulmonary vascular bed and increased pulmonary vascular resistance, potentially culminating in pulmonary hypertension.

177
Q

Which of the following markers is not commonly associated with sarcoidosis?
A) Serum amyloid A
B) Adenosine deaminase
C) Serum soluble IL2 receptor
D) C-reactive protein (CRP)

A

Correct Answer: D) C-reactive protein (CRP)

Rationale: While CRP is a marker of inflammation, it is not specifically associated with sarcoidosis. Serum amyloid A, adenosine deaminase, and serum soluble IL2 receptor are markers that can be elevated in sarcoidosis and help in supporting the diagnosis.

178
Q

Hypersensitivity pneumonitis is characterized by what type of infiltrates in the lungs on imaging?
A) Mucoid impaction
B) Cavitary lesions
C) Ground-glass opacities
D) Pleural effusions

A

Correct Answer: C) Ground-glass opacities

Rationale: Hypersensitivity pneumonitis, on computed tomography (CT) scans, typically shows ground-glass opacities, especially in the mid to upper lung zones, indicating inflammation and interstitial changes due to the immune response to inhaled antigens.

179
Q

Which of the following is not a typical presentation timeframe for symptoms after antigen exposure in hypersensitivity pneumonitis?
A) 4-6 hours
B) 12-24 hours
C) 48-72 hours
D) 1-4 hours

A

Correct Answer: D) 1-4 hours

Rationale: The symptoms of hypersensitivity pneumonitis characteristically appear 4-6 hours after exposure to the antigen. Symptoms developing within 1-4 hours are not typical for this condition and may suggest a different etiology.

180
Q

What is the mainstay treatment for hypersensitivity pneumonitis?
A) Broad-spectrum antibiotics
B) Antigen avoidance and glucocorticoids
C) Beta-agonist inhalers
D) Antiviral therapy

A

Correct Answer: B) Antigen avoidance and glucocorticoids

Rationale: The primary treatment for hypersensitivity pneumonitis involves avoiding further exposure to the inciting antigen and the use of glucocorticoids to reduce the inflammatory response in the lungs. In severe cases, a lung transplant may be considered.

181
Q

Repeated episodes of hypersensitivity pneumonitis can lead to which long-term pulmonary complication?
A) Emphysema
B) Asthma
C) Pulmonary fibrosis
D) Chronic bronchitis

A

Correct Answer: C) Pulmonary fibrosis

Rationale: Chronic and repeated episodes of hypersensitivity pneumonitis can cause progressive lung damage and scarring, leading to pulmonary fibrosis. This can result in chronic respiratory failure and other serious complications.

182
Q

In the acute presentation of hypersensitivity pneumonitis, which hematologic findings are typically observed after exposure to the offending antigen?
A) Thrombocytopenia and anemia
B) Leukocytosis and eosinophilia
C) Neutropenia and lymphopenia
D) Basophilia and monocytosis

A

Correct Answer: B) Leukocytosis and eosinophilia

Rationale: After inhalation of the antigen causing hypersensitivity pneumonitis, there is often an acute inflammatory response characterized by leukocytosis (an increase in white blood cells) and eosinophilia (an increase in eosinophils). These findings reflect the immune system’s response to the inhaled allergens.

182
Q

What is the significance of eosinophilia in the context of hypersensitivity pneumonitis?
A) It is indicative of bacterial infection.
B) It suggests viral etiology of pneumonitis.
C) It denotes an allergic response contributing to inflammation.
D) It indicates the presence of parasitic infestation.

A

Correct Answer: C) It denotes an allergic response contributing to inflammation.

Rationale: Eosinophilia in the context of hypersensitivity pneumonitis is significant as it denotes an allergic or hypersensitivity reaction within the lungs contributing to the overall inflammatory process.

183
Q

Pulmonary Langerhans Cell Histiocytosis is primarily associated with which of the following risk factors?
A) Occupational asbestos exposure
B) Tobacco smoking
C) Genetic predisposition
D) Environmental pollution

A

Correct Answer: B) Tobacco smoking

Rationale: Pulmonary Langerhans Cell Histiocytosis has a strong association with tobacco smoking, which is considered the most significant risk factor for the development of this disease.

184
Q

What radiographic feature on CT is indicative of Pulmonary Langerhans Cell Histiocytosis?

A) Pleural effusions
B) Enlarged hilar lymph nodes
C) Cysts or honeycombing in the upper lung zones with costophrenic sparing
D) Diffuse alveolar damage

A

Correct Answer: C) Cysts or honeycombing in the upper lung zones with costophrenic sparing

Rationale: CT scans of patients with Pulmonary Langerhans Cell Histiocytosis typically show cysts or honeycombing patterns in the upper zones of the lungs, with sparing of the costophrenic angles, which is a distinctive feature that can be diagnostic of the disease.

185
Q

Lung biopsy in Pulmonary Langerhans Cell Histiocytosis characteristically shows inflammatory lesions with a distinct population of cells. Which of the following cells is most indicative of this disease?
A) Macrophages
B) Neutrophils
C) Langerhans cells
D) Plasma cells

A

Correct Answer: C) Langerhans cells

Rationale: The histological hallmark of Pulmonary Langerhans Cell Histiocytosis is the presence of Langerhans cells within the inflammatory lesions around the bronchioles. These cells can be identified by their characteristic appearance and immunohistochemical staining.

186
Q

The primary treatment for Pulmonary Langerhans Cell Histiocytosis includes which of the following?
A) Long-term oxygen therapy
B) Smoking cessation and systemic glucocorticoids
C) Antitubercular therapy
D) Aggressive chemotherapy

A

Correct Answer: B) Smoking cessation and systemic glucocorticoids

Rationale: The cornerstone of treatment for Pulmonary Langerhans Cell Histiocytosis involves smoking cessation to halt the progression of the disease, along with the use of systemic glucocorticoids to reduce inflammation and provide symptomatic relief.

187
Q

What radiographic finding is characteristic of Pulmonary Alveolar Proteinosis (PAP) on a chest X-ray?
A) Pleural plaques
B) Hilar lymphadenopathy
C) Batwing distribution of alveolar opacities
D) Hyperinflation of lung fields

A

Correct Answer: C) Batwing distribution of alveolar opacities

Rationale: The batwing or butterfly pattern distribution of alveolar opacities, typically in the middle and lower lung zones, is a characteristic radiographic feature of PAP seen on a chest X-ray.

188
Q

The treatment of severe cases of Pulmonary Alveolar Proteinosis (PAP) involves:
A) Administering high doses of antibiotics
B) Performing whole-lung lavage under general anesthesia
C) Using long-term corticosteroid therapy
D) Immediate lung transplant

A

Correct Answer: B) Performing whole-lung lavage under general anesthesia

Rationale: Whole-lung lavage under general anesthesia is the treatment of choice for severe cases of PAP to remove the accumulated proteinaceous material in the alveoli and improve macrophage function, thereby enhancing gas exchange and alleviating symptoms.

189
Q

During anesthesia for lung lavage in a patient with PAP, which airway management technique is critical for optimizing oxygenation?
A) Single-lumen endotracheal tube with bronchial blockers
B) Double-lumen endotracheal tube (DLT)
C) High-frequency oscillatory ventilation
D) Non-invasive ventilation with CPAP

A

Correct Answer: B) Double-lumen endotracheal tube (DLT)

Rationale: The use of a double-lumen endotracheal tube (DLT) allows for selective isolation and ventilation of one lung while the other is being lavaged, which is crucial for maintaining oxygenation during the procedure in patients with PAP who are at risk of hypoxemia.

190
Q

Pulmonary Alveolar Proteinosis (PAP) is associated with which of the following systemic conditions?
A) Chronic obstructive pulmonary disease (COPD)
B) Pulmonary hypertension
C) Acquired immunodeficiency syndrome (AIDS)
D) Cystic fibrosis

A

Correct Answer: C) Acquired immunodeficiency syndrome (AIDS)

Rationale: PAP may occur independently or in association with other conditions, including immunodeficiency states such as AIDS, exposure to certain chemicals or dust, or following chemotherapy, reflecting a disruption in normal alveolar macrophage function.

191
Q

Lymphangioleiomyomatosis (LAM) predominantly affects which population?
A) Men of advanced age
B) Postmenopausal women
C) Women of reproductive age
D) Children

A

Correct Answer: C) Women of reproductive age

Rationale: Lymphangioleiomyomatosis is a rare multisystem disorder that primarily occurs in women of reproductive age, characterized by the proliferation of smooth muscle cells, which can affect the lungs, lymphatics, and blood vessels.

192
Q

What pulmonary function test (PFT) pattern is typically seen in patients with Lymphangioleiomyomatosis (LAM)?
A) Isolated restrictive pattern
B) Isolated obstructive pattern
C) Combined restrictive and obstructive disease with decreased diffusing capacity
D) Normal pattern with increased diffusing capacity

A

Correct Answer: C) Combined restrictive and obstructive disease with decreased diffusing capacity

Rationale: Patients with LAM may exhibit a mixed restrictive and obstructive pattern on pulmonary function tests, often accompanied by a decreased diffusing capacity due to the disease’s impact on lung architecture and function.

193
Q

Which treatment has been indicated for symptomatic patients with Lymphangioleiomyomatosis (LAM) to help slow disease progression?
A) Beta-agonists
B) Sirolimus (immunosuppressive therapy)
C) Antibiotics
D) Anticoagulation therapy

A

Correct Answer: B) Sirolimus (immunosuppressive therapy)

Rationale: Sirolimus, an immunosuppressive agent, has been shown to be beneficial in the treatment of LAM by slowing disease progression in symptomatic patients, likely due to its inhibitory effect on the mTOR pathway, which is involved in cell growth and proliferation.

194
Q

The clinical presentation of Lymphangioleiomyomatosis (LAM) may include all of the following except:
A) Hemoptysis
B) Recurrent pneumothorax
C) Pleural effusions
D) Chronic productive cough

A

Correct Answer: D) Chronic productive cough

Rationale: LAM is characterized by progressive dyspnea, hemoptysis, recurrent pneumothorax, and pleural effusions. A chronic productive cough is not typically a feature of LAM and is more commonly associated with conditions involving chronic bronchial inflammation or infection.

195
Q

What changes in lung volumes are typically associated with aging?

A) Increased residual volume (RV) and increased vital capacity (VC)
B) Decreased RV and increased VC
C) Increased RV and decreased VC
D) Decreased RV and decreased VC

A

Correct Answer: C) Increased RV and decreased VC

Rationale: As a person ages, physiological changes in the lungs and chest wall, such as decreased chest wall compliance and decreased elastic recoil, lead to an increased residual volume (RV) due to less effective exhalation, and decreased vital capacity (VC) due to reduced total lung capacity.

196
Q

Which thoracic change associated with aging can decrease the efficiency of the diaphragm?
A) Flattening of the diaphragm
B) Decrease in the kyphosis of the thoracic spine
C) Increase in the anteroposterior (AP) diameter of the chest
D) Decrease in the AP diameter of the chest

A

Correct Answer: C) Increase in the anteroposterior (AP) diameter of the chest

Rationale: An increase in the anteroposterior (AP) diameter of the chest, often due to kyphosis or changes in thoracic spine curvature associated with aging, can alter the orientation and thus the mechanical efficiency of the diaphragm during breathing.

197
Q

In the context of aging, how is the functional residual capacity (FRC) of the lungs affected?
A) FRC decreases due to reduced lung compliance
B) FRC increases as a compensatory mechanism to maintain oxygenation
C) FRC remains unchanged but the respiratory rate increases
D) FRC decreases due to the increased efficiency of respiratory muscles

A

Correct Answer: B) FRC increases as a compensatory mechanism to maintain oxygenation

Rationale: In older adults, the functional residual capacity (FRC) often increases due to changes in lung and chest wall mechanics. Geriatric patients tend to breathe at higher lung volumes to compensate for the decreased elasticity and compliance of the respiratory system.

198
Q

What pulmonary function test (PFT) parameters commonly show a decline with advancing age?

A) Forced vital capacity (FVC) and forced expiratory volume in 1 second (FEV1)
B) Peak expiratory flow (PEF) and maximal voluntary ventilation (MVV)
C) Total lung capacity (TLC) and peak inspiratory pressure (PIP)
D) Forced inspiratory volume in 1 second (FIV1) and maximal inspiratory pressure (MIP)

A

Correct Answer: A) Forced vital capacity (FVC) and forced expiratory volume in 1 second (FEV1)

Rationale: Aging is commonly associated with a decline in both FEV1 and FVC on pulmonary function testing, reflecting changes in lung mechanics such as decreased compliance and elastic recoil. This decline can be more pronounced in patients with increased airway reactivity.

199
Q

Which deformities are commonly associated with chronic extrinsic restrictive lung disease (RLD)?

A) Pectus excavatum and pectus carinatum
B) Sternal fractures and rib contusions
C) Ankylosing spondylitis, flail chest, scoliosis, and kyphosis
D) Costochondritis and Tietze syndrome

A

Correct Answer: C) Ankylosing spondylitis, flail chest, scoliosis, and kyphosis

Rationale: Chronic extrinsic RLD is often related to structural deformities of the thoracic cage such as ankylosing spondylitis, flail chest, scoliosis, and kyphosis, which can interfere with normal lung expansion and mechanics.

200
Q

How does chronic extrinsic restrictive lung disease affect the work of breathing?

A) It decreases due to more rigid chest wall mechanics
B) It increases due to abnormal mechanics and increased airway resistance
C) It remains unchanged but respiratory muscle endurance decreases
D) It decreases due to compensatory hyperinflation

A

Correct Answer: B) It increases due to abnormal mechanics and increased airway resistance

Rationale: In chronic extrinsic RLD, the work of breathing is increased due to the abnormal chest wall mechanics that result in increased airway resistance and decreased lung volumes, making the effort of breathing more labor-intensive.

201
Q

What can thoracic deformity lead to in terms of cardiopulmonary function?
A) Improved cardiac output due to reduced intrathoracic pressure
B) Left ventricular hypertrophy due to increased workload
C) Right ventricular dysfunction due to compression of pulmonary vasculature
D) Decreased venous return due to high intrathoracic pressures

A

Correct Answer: C) Right ventricular dysfunction due to compression of pulmonary vasculature

Rationale: Thoracic deformities can lead to compression of the pulmonary vasculature, increasing the resistance against which the right ventricle must pump. This can eventually lead to right ventricular dysfunction.

202
Q

What is a common complication of chronic extrinsic restrictive lung diseases due to a poor cough mechanism?
A) Atelectasis
B) Pulmonary edema
C) Recurrent pulmonary infections
D) Chronic hypoxemia

A

Correct Answer: C) Recurrent pulmonary infections

Rationale: A reduced ability to cough effectively, which is common in chronic extrinsic restrictive lung diseases, leads to an inability to clear secretions from the respiratory tract, predisposing patients to recurrent pulmonary infections.

203
Q

What are the two types of costovertebral skeletal deformities commonly associated with chronic extrinsic restrictive lung disease?
A) Osteoporosis and osteoarthritis
B) Scoliosis and lordosis
C) Scoliosis and kyphosis
D) Kyphosis and lordosis

A

Correct Answer: C) Scoliosis and kyphosis

Rationale: Scoliosis and kyphosis are the two types of spinal deformities that can lead to chronic extrinsic restrictive lung disease due to their impact on the thoracic cage and its capacity to expand, often resulting in restrictive lung impairment.

204
Q

Patients with kyphoscoliosis related to neuromuscular disorders tend to have what compared to those with idiopathic kyphoscoliosis?

A) Less respiratory compromise
B) Similar respiratory compromise
C) More respiratory compromise
D) Only obstructive impairment without restrictive changes

A

Correct Answer: C) More respiratory compromise

Rationale: Patients with kyphoscoliosis due to neuromuscular disorders typically experience more severe respiratory compromise than those with idiopathic kyphoscoliosis because the neuromuscular etiology often affects both the structure and function of respiratory muscles, leading to a more pronounced impairment.

205
Q

The severity of respiratory compromise in kyphoscoliosis is most closely related to:
A) The age at which the condition develops
B) The presence of underlying cardiac conditions
C) The degree of spinal curvature
D) The patient’s body mass index (BMI)

A

Correct Answer: C) The degree of spinal curvature

Rationale: The degree of respiratory compromise in patients with kyphoscoliosis is directly correlated with the severity of the spinal curvature. Greater curvature can more significantly restrict lung expansion, leading to reduced ventilatory capacity and increased work of breathing.

206
Q

When does kyphoscoliosis typically begin to develop?
A) In infancy
B) In late childhood/early adolescence
C) In young adulthood
D) After middle age

A

Correct Answer: B) In late childhood/early adolescence

Rationale: Kyphoscoliosis commonly begins in late childhood or early adolescence, which are periods of rapid skeletal growth. This is when the signs and symptoms of the deformity and the associated respiratory compromise often start to become evident.

207
Q

What is the common name for the deformity of the sternum known as pectus carinatum?
A) Funnel chest
B) Barrel chest
C) Pigeon chest
D) Ravitch syndrome

A

Correct Answer: C) Pigeon chest

Rationale: Pectus carinatum is commonly referred to as “pigeon chest.” It is characterized by a protrusion of the sternum and ribs and, while often considered a cosmetic issue, can be associated with respiratory symptoms.

208
Q

Which thoracic injury is characterized by paradoxical inward movement of part of the rib cage during inspiration?
A) Sternum fracture
B) Flail chest
C) Costochondritis
D) Tietze syndrome

A

Correct Answer: B) Flail chest

Rationale: Flail chest occurs due to multiple rib fractures causing a segment of the chest wall to become detached from the rest of the thoracic cage. This results in paradoxical movements during breathing, where the segment moves inward on inspiration and outward on expiration.

209
Q

What is a primary symptom of a flail chest?
A) Cough productive of clear sputum
B) Sharp chest pain exacerbated by arm movement
C) Increased work of breathing and pain
D) Wheezing on exertion

A

Correct Answer: C) Increased work of breathing and pain

Rationale: Patients with flail chest experience increased work of breathing due to the instability of the chest wall and pain due to the multiple rib fractures. This condition can lead to respiratory compromise and is considered a medical emergency.

210
Q

What is the mainstay treatment for flail chest?
A) Analgesics and chest physiotherapy
B) Nonsteroidal anti-inflammatory drugs (NSAIDs) and bed rest
C) Positive pressure ventilation until stabilization
D) Immediate surgical fixation of rib fractures

A

Correct Answer: C) Positive pressure ventilation until stabilization

Rationale: Treatment for flail chest typically involves the use of positive pressure ventilation to stabilize the affected segment of the chest wall, improve ventilation, and support breathing while the underlying fractures heal.

211
Q

Which imaging modality is preferred for the diagnosis of a pleural effusion?
A) Chest X-ray (CXR)
B) Computed tomography (CT)
C) Bedside ultrasound (US)
D) Magnetic resonance imaging (MRI)

A

Correct Answer: C) Bedside ultrasound (US)

Rationale: Bedside ultrasound is the preferred method for diagnosing pleural effusion as it is highly sensitive, can be performed at the bedside, allows for the evaluation of fluid characteristics, and can guide therapeutic procedures if needed.

212
Q

Idiopathic spontaneous pneumothorax (ptx) is most commonly seen in individuals with which characteristics?

A) Overweight males aged 50-60
B) Tall, thin men aged 20-40
C) Women of reproductive age
D) Elderly females with osteoporosis

A

Correct Answer: B) Tall, thin men aged 20-40

Rationale: Idiopathic spontaneous pneumothorax commonly occurs in tall, thin men between the ages of 20-40 and is often attributed to the rupture of apical subpleural blebs.

213
Q

What is the immediate treatment for a tension pneumothorax?
A) Antibiotic therapy
B) High-flow oxygen and observation
C) Thoracotomy and surgical intervention
D) Needle decompression or placement of a chest tube

A

Correct Answer: D) Needle decompression or placement of a chest tube

Rationale: A tension pneumothorax is a life-threatening condition that requires immediate intervention to relieve the pressure in the pleural space. This is typically achieved by needle decompression or insertion of a chest tube in the second anterior intercostal space, which can be lifesaving.

214
Q

In a tension pneumothorax, what clinical finding may be observed regarding the position of the trachea?
A) No change in position
B) Tracheal deviation towards the side of the pneumothorax
C) Tracheal deviation away from the side of the pneumothorax
D) Tracheal compression and collapse

A

Correct Answer: C) Tracheal deviation away from the side of the pneumothorax

Rationale: In a tension pneumothorax, the build-up of pressure in the pleural space can push the mediastinum and trachea away from the affected side, which is an important clinical sign indicating the severity of the condition.

215
Q

What is often the underlying cause of acute mediastinitis?
A) Viral infection
B) Myocardial infarction
C) Bacterial contamination following esophageal perforation
D) Pulmonary embolism

A

Correct Answer: C) Bacterial contamination following esophageal perforation

Rationale: Acute mediastinitis is commonly caused by bacterial contamination, which can occur as a result of esophageal perforation. This serious condition is associated with symptoms such as chest pain and fever and requires prompt treatment with broad-spectrum antibiotics and surgical drainage.

216
Q

Which type of mediastinal mass is most commonly found in the anterior mediastinum?
A) Neurogenic tumors
B) Thymomas
C) Tracheal masses
D) Esophageal diverticula

A

Correct Answer: B) Thymomas

Rationale: Thymomas are the most common masses found in the anterior mediastinum, comprising about 20% of mediastinal tumors. Other anterior mediastinal masses can include germ cell tumors, lymphomas, intrathoracic thyroid tissue, and parathyroid lesions.

217
Q

What surgical procedure may be considered for symptomatic patients with pleural fibrosis?
A) Pleurodesis
B) Lung transplant
C) Decortication
D) Thoracentesis

A

Correct Answer: C) Decortication

Rationale: Surgical decortication may be considered for patients with pleural fibrosis who are symptomatic. This procedure involves the removal of thick, fibrous tissue to allow better lung expansion and function.

218
Q

Posterior mediastinal masses can include all the following EXCEPT:
A) Meningoceles
B) Descending aortic aneurysms
C) Intrathoracic thyroid tissue
D) Neurogenic tumors

A

Correct Answer: C) Intrathoracic thyroid tissue

Rationale: Intrathoracic thyroid tissue is generally located in the anterior mediastinum. Posterior mediastinal masses more commonly include neurogenic tumors and cysts, meningoceles, lymphomas, descending aortic aneurysms, and esophageal diverticula and neoplasms.

219
Q

What is the purpose of measuring a flow-volume loop in the preoperative evaluation for a mediastinal mass?
A) To assess for restrictive lung disease
B) To determine the patient’s cardiac function
C) To evaluate for airway compression
D) To identify the type of mediastinal mass

A

Correct Answer: C) To evaluate for airway compression

Rationale: A flow-volume loop is measured preoperatively to assess for the presence and degree of airway compression by a mediastinal mass, as it can reveal extrathoracic or intrathoracic obstruction.

220
Q

Which diagnostic procedure is useful for determining the extent of airway obstruction due to a mediastinal mass?
A) Spirometry
B) Echocardiogram
C) Fiberoptic bronchoscopy
D) Arterial blood gas analysis

A

Correct Answer: C) Fiberoptic bronchoscopy

Rationale: Fiberoptic bronchoscopy is a valuable tool for directly visualizing the airways and assessing the extent and nature of airway obstruction caused by a mediastinal mass.

221
Q

What should be considered preoperatively to potentially reduce the size of a mediastinal mass?
A) High-dose steroid therapy
B) Antibiotic therapy
C) Mass radiation
D) Immediate surgical excision

A

Correct Answer: C) Mass radiation

Rationale: Preoperative radiation therapy may be considered to decrease the size of a mediastinal mass, particularly if it is causing symptomatic compression of the trachea or other critical structures.

222
Q

What does the degree of preoperative pulmonary compromise indicate about the potential intraoperative risks during anesthesia for a mediastinal mass?

A) It correlates strongly with the risk of intraoperative respiratory compromise.
B) It is not indicative of the degree of respiratory compromise that may be encountered during anesthesia.
C) It suggests the need for invasive mechanical ventilation during surgery.
D) It determines the surgical technique to be employed.

A

Correct Answer: B) It is not indicative of the degree of respiratory compromise that may be encountered during anesthesia.

Rationale: The severity of preoperative pulmonary compromise does not always correlate with the degree of respiratory compromise that can be encountered during anesthesia, as even asymptomatic patients can develop severe airway obstruction when anesthetized.

223
Q

In symptomatic patients requiring a diagnostic tissue biopsy of a mediastinal mass, which approach is preferred?
A) Thoracotomy
B) Transthoracic needle biopsy
C) Local anesthetic (LA) technique
D) General anesthesia (GA) with intubation

A

Correct Answer: C) Local anesthetic (LA) technique

Rationale: For symptomatic patients who require a tissue biopsy of a mediastinal mass, a local anesthetic technique is preferred to avoid the risks associated with general anesthesia, such as exacerbation of airway compression or respiratory compromise.

224
Q

What genetic pattern is associated with Asphyxiating thoracic dystrophy, also known as Jeune syndrome?
A) Autosomal dominant
B) Autosomal recessive
C) X-linked recessive
D) Mitochondrial inheritance

A

Correct Answer: B) Autosomal recessive

Thoracic cause of restrictive lung disease.

Rationale: Asphyxiating thoracic dystrophy (Jeune syndrome) is an autosomal recessive disorder, meaning that it typically requires two copies of the mutated gene, one from each parent, for the disorder to manifest in the offspring.

225
Q

Fibrodysplasia ossificans progressiva is characterized by which type of genetic variation?
A) Mutation in the COL1A1 gene
B) Variation in bone morphogenetic protein (BMP) type 1
C) Abnormality in the fibroblast growth factor gene
D) Deficiency in the mineralization process of bones

A

Correct Answer: B) Variation in bone morphogenetic protein (BMP) type 1

Rationale: Fibrodysplasia ossificans progressiva is a hereditary disorder caused by a genetic variation in the bone morphogenetic protein (BMP) type 1, leading to abnormal ossification of connective tissue.

226
Q

Which syndrome is characterized by the absence of pectoral muscles on one side of the body?
A) Marfan syndrome
B) Turner syndrome
C) Poland syndrome
D) Ehlers-Danlos syndrome

A

Correct Answer: C) Poland syndrome

Rationale: Poland syndrome is characterized by the partial or complete absence of the pectoral muscles on one side of the body, which can lead to asymmetry and potentially paradoxical respiratory motion due to the absence of multiple ribs.

227
Q

Patients with Poland syndrome may exhibit what type of respiratory motion?
A) Synchronous diaphragmatic flutter
B) Consistent bilateral chest expansion
C) Paradoxical respiratory motion
D) Unilateral diaphragmatic paralysis

A

Correct Answer: C) Paradoxical respiratory motion

Rationale: Due to the absence of the pectoral muscles and possibly multiple ribs, patients with Poland syndrome may display paradoxical respiratory motion, where the chest wall moves inward during inspiration instead of outward.

228
Q

What aspect of respiratory function is particularly affected by neuromuscular disorders in restrictive lung disease?
A) Lung compliance
B) Gas exchange efficiency
C) Airway resistance
D) Respiratory muscle strength

A

Correct Answer: D) Respiratory muscle strength

Rationale: Neuromuscular disorders can impair the central nervous system’s input to the respiratory muscles, leading to weakened respiratory muscles and an inability to generate normal respiratory pressures, which is a hallmark of restrictive lung defects in these conditions.

229
Q

What is the consequence of expiratory muscle weakness in patients with neuromuscular disorders?
A) Improved cough effectiveness
B) Reduced ability to cough effectively
C) Decreased respiratory rate
D) Increased lung compliance

A

Correct Answer: B) Reduced ability to cough effectively

Rationale: Expiratory muscle weakness in neuromuscular disorders hampers the ability to generate sufficient airflow during coughing, leading to an ineffective cough and potential difficulty in clearing airway secretions.

230
Q

During which state are patients with severe neuromuscular disorders more able to maintain adequate ventilation?
A) Sleep
B) Sedation
C) Wakefulness
D) Relaxation

A

Correct Answer: C) Wakefulness

Rationale: Patients with severe neuromuscular disorders often depend on their state of wakefulness to maintain adequate ventilation, as the decreased muscle tone during sleep can exacerbate ventilatory insufficiency.

231
Q

What percentage of patients with Guillain-Barré syndrome typically require mechanical ventilation?
A) 5-10%
B) 20-25%
C) 30-35%
D) 40-45%

A

Correct Answer: B) 20-25%

Rationale: About 20-25% of patients with Guillain-Barré syndrome may require mechanical ventilation (2 months) due to the disease’s effect on the respiratory muscles, which can lead to respiratory failure.

232
Q

What can develop during sleep in patients with neuromuscular disorders contributing to the development of cor pulmonale?
A) Hyperoxia and hypocapnia
B) Hypoxemia and hypercapnia
C) Normoxemia and normocapnia
D) Polycythemia and alkalosis

A

Correct Answer: B) Hypoxemia and hypercapnia

Rationale: During sleep, patients with neuromuscular disorders may experience hypoventilation leading to hypoxemia (low oxygen levels) and hypercapnia (elevated carbon dioxide levels), which can contribute to increased pulmonary arterial pressure and the development of cor pulmonale (right-sided heart failure).

233
Q

Patients with Myasthenia gravis have an altered response to which type of medication?
A) Beta-blockers
B) Non-depolarizing neuromuscular blockers (ND-NMBs)
C) Antibiotics
D) Diuretics

A

Correct Answer: B) Non-depolarizing neuromuscular blockers (ND-NMBs)

Rationale: Patients with Myasthenia gravis are particularly sensitive to the effects of non-depolarizing neuromuscular blockers (ND-NMBs) due to their underlying neuromuscular transmission defect and are often resistant to the effects of succinylcholine.

234
Q

Which respiratory complication is a patient with Muscular Dystrophy prone to due to inspiratory muscle weakness?
A) Spontaneous pneumothorax
B) Chronic bronchitis
C) Chronic alveolar hypoventilation
D) Asthma

A

Correct Answer: C) Chronic alveolar hypoventilation

Rationale: Chronic alveolar hypoventilation occurs in Muscular Dystrophy mainly due to inspiratory muscle weakness, leading to inadequate ventilation and the risk of respiratory failure, especially during sleep

235
Q

What is the implication of expiratory muscle weakness in patients with Muscular Dystrophy?
A) Enhanced cough efficiency
B) Impaired cough and risk of aspiration
C) Increased vital capacity
D) Reduced risk of pulmonary infections

A

Correct Answer: B) Impaired cough and risk of aspiration

Rationale: Expiratory muscle weakness in Muscular Dystrophy impairs the ability to cough effectively, increasing the risk of pulmonary aspiration and subsequent respiratory complications.

236
Q

Why should CNS depressant drugs be used cautiously in patients with Muscular Dystrophy?
A) They can exacerbate muscle weakness
B) They can induce seizure activity
C) They can cause acute renal failure
D) They can lead to hypersensitivity reactions

A

Correct Answer: A) They can exacerbate muscle weakness

Rationale: CNS depressant drugs should be minimized or avoided in patients with Muscular Dystrophy because they can further depress respiratory drive and exacerbate already compromised respiratory muscle function.

237
Q

At what level of spinal cord injury is diaphragmatic breathing primarily preserved?
A) Above C4
B) Below T4
C) At T6
D) Above L1

A

Correct Answer: B) Below T4

Rationale: In quadriplegic patients with a spinal cord injury below T4, diaphragmatic breathing is maintained because the phrenic nerve, which innervates the diaphragm, originates from the C3-C5 spinal segments. Therefore, injuries below T4 typically spare the diaphragm.

238
Q

What respiratory function is compromised in patients with high spinal cord injuries due to diaphragmatic paralysis?
A) Inspiration
B) Expiration
C) Coughing
D) Gas exchange

A

Correct Answer: C) Coughing

Rationale: Since the diaphragm is active only during inspiration, high spinal cord injuries lead to an inability to cough effectively because coughing relies on expiratory muscle action which is affected by the injury.

239
Q

Which type of motion is characteristic of diaphragmatic breathing in patients with high spinal cord injuries?
A) Symmetrical chest wall expansion
B) Paradoxical inward motion of the upper thorax during inspiration
C) Increased tidal volume (TV) with each breath
D) Active abdominal muscle contraction during inspiration

A

Correct Answer: B) Paradoxical inward motion of the upper thorax during inspiration

Rationale: With diaphragmatic breathing, particularly in the context of high spinal cord injuries, there is often a paradoxical motion where the upper thorax moves inward during inspiration due to the lack of intercostal muscle function, resulting in diminished TV.

240
Q

What drug class can be used to treat mild bronchial constriction in quadriplegic patients?
A) Beta-agonists
B) Corticosteroids
C) Anticholinergic bronchodilators
D) Non-steroidal anti-inflammatory drugs (NSAIDs)

A

Correct Answer: C) Anticholinergic bronchodilators

Rationale: Anticholinergic bronchodilators can be used to reverse mild bronchial constriction in quadriplegic patients. This bronchial constriction is due to unopposed parasympathetic tone as a result of the lack of sympathetic innervation from the spinal cord.

241
Q

What lung volume measurements are commonly decreased in individuals with obesity?
A) Forced expiratory volume in 1 second (FEV1), forced vital capacity (FVC), functional residual capacity (FRC), expiratory reserve volume (ERV)
B) Total lung capacity (TLC), peak expiratory flow (PEF), inspiratory reserve volume (IRV), inspiratory capacity (IC)
C) Tidal volume (TV), residual volume (RV), maximal voluntary ventilation (MVV), peak inspiratory flow (PIF)
D) Slow vital capacity (SVC), minute ventilation (MV), vital capacity (VC), forced inspiratory flow (FIF)

A

Correct Answer: A) Forced expiratory volume in 1 second (FEV1), forced vital capacity (FVC), functional residual capacity (FRC), expiratory reserve volume (ERV)

Rationale: Obesity is associated with decreases in several lung volumes including FEV1, FVC, FRC, and ERV due to the increased chest wall mass, which limits expansion and increases the work of breathing.

242
Q

In patients with extreme obesity, the functional residual capacity (FRC) may become compromised in what
way?

A) FRC may become so reduced that it falls below the closing volume
B) FRC may exceed the closing volume and approach the residual volume (RV)
C) FRC may increase to exceed the total lung capacity (TLC)
D) FRC remains unchanged regardless of the degree of obesity

A

Correct Answer: B) FRC may exceed the closing volume and approach the residual volume (RV) - sounds like losing the ERV

Rationale: In extreme cases of clinical obesity, FRC may increase to levels that exceed the closing volume of the lungs, which is the volume at which small airways begin to collapse, and can approach the residual volume (RV), the volume of air remaining in the lungs after a maximal exhalation.

243
Q

How does obesity typically affect the FEV1:FVC ratio?
A) It decreases significantly.
B) It increases beyond normal limits.
C) It is usually preserved.
D) It becomes inverted.

A

Correct Answer: C) It is usually preserved.

Rationale: Despite reductions in absolute values of FEV1 and FVC in obesity, their ratio (FEV1:FVC) is typically preserved, indicating that obesity affects both volumes proportionally and does not typically produce an obstructive ventilatory defect.

244
Q

What is the role of adipocytes in the pathophysiology of obesity-related respiratory disorders?

A) They increase oxygenation and reduce airway inflammation.
B) They release adipocytokines that contribute to systemic inflammation and respiratory disorders.
C) They improve lung compliance and decrease the risk of obstructive sleep apnea (OSA).
D) They contribute to someone being a fat fuck

A

Correct Answer: B) They release adipocytokines that contribute to systemic inflammation and respiratory disorders.

Rationale: Adipocytes release adipocytokines, which are involved in systemic inflammation and can contribute to the development of various respiratory disorders in obesity, such as hypoxemia, OSA, obesity hypoventilation syndrome, and potentially COPD.

244
Q

What effect does adipose buildup in the anterior abdominal wall have on respiratory mechanics?
A) Enhances diaphragmatic movement
B) Diminishes basal lung expansion and causes closure of peripheral lung units
C) Increases lung compliance
D) Promotes deeper inspiratory efforts

A

Correct Answer: B) Diminishes basal lung expansion and causes closure of peripheral lung units

Rationale: The accumulation of adipose tissue in the anterior abdominal wall in individuals with obesity can limit the movement of the diaphragm, diminishing basal lung expansion and leading to closure of peripheral lung units, especially at lower lung volumes.

245
Q

How does pregnancy most commonly affect the respiratory system?
A) By leading to obstructive lung physiology
B) By causing a restrictive lung physiology
C) By significantly increasing lung compliance
D) By decreasing the anteroposterior diameter of the chest wall

A

Correct Answer: B) By causing a restrictive lung physiology

Rationale: Pregnancy can lead to a restrictive lung physiology as the enlarging uterus pushes the diaphragm upward, which reduces the volume of the lungs during inhalation. Additionally, the expanding rib cage and stretching of the lower rib cage ligaments due to hormones like relaxin contribute to this change.

246
Q

What causes the expansion of the rib cage circumference during pregnancy?
A) Decreased levels of progesterone
B) Increased levels of relaxin causing stretching of the lower rib cage ligaments
C) Reduction in the subcostal angle
D) Contraction of the diaphragm

A

Correct Answer: B) Increased levels of relaxin causing stretching of the lower rib cage ligaments

Rationale: The hormone relaxin, which increases during pregnancy, causes the ligaments of the pelvis and the lower rib cage to stretch, leading to an increase in the anteroposterior and transverse diameters of the chest walls and an expanded rib cage circumference.

247
Q

When do the changes in the chest wall configuration related to pregnancy usually peak?
A) At the onset of labor
B) At the 37th week of pregnancy
C) Immediately after delivery
D) During the first trimester

A

Correct Answer: B) At the 37th week of pregnancy

Rationale: The changes in chest wall configuration due to pregnancy, such as the widening of the subcostal angle and increase in chest circumference, typically peak at the 37th week of pregnancy.

248
Q

How long after delivery does it take for the chest wall configuration to normalize postpartum?
A) Immediately after delivery
B) Within the first month postpartum
C) About 6 months postpartum
D) One year postpartum

A

Correct Answer: C) About 6 months postpartum

Rationale: The chest wall configuration generally normalizes about 6 months after delivery, with the exception of the subcostal angle, which remains wider by about 20%.

249
Q

By approximately how much does the diaphragm move cephalad due to the enlarging uterus during pregnancy?
A) 2 cm
B) 4 cm
C) 6 cm
D) 8 cm

A

Correct Answer: B) 4 cm

Rationale: The enlarging uterus during pregnancy pushes the diaphragm upward by about 4 cm, contributing to the changes in lung volumes and mechanics during pregnancy.

250
Q

Why should drugs with prolonged respiratory depressant effects be avoided in patients with restrictive lung disease (RLD)?
A) They can cause an acute asthma attack.
B) They can lead to hyperventilation and respiratory alkalosis.
C) They can exacerbate existing hypoventilation and lead to further respiratory compromise.
D) They have no effect on the central respiratory drive.

A

Correct Answer: C) They can exacerbate existing hypoventilation and lead to further respiratory compromise.

Rationale: In RLD, where lung compliance is already reduced, drugs with prolonged respiratory depressant effects can further diminish respiratory drive and exacerbate hypoventilation, worsening the patient’s respiratory status.

251
Q

What is often required intraoperatively to manage patients with restrictive lung disease effectively?
A) Manual ventilation only
B) Mechanical ventilation with reduced inspiratory pressures
C) Intraoperative mechanical ventilation to facilitate optimal oxygenation
D) Spontaneous breathing without any assistance

A

Correct Answer: C) Intraoperative mechanical ventilation to facilitate optimal oxygenation

Rationale: Intraoperative mechanical ventilation is important in managing patients with RLD, as it can help maintain optimal oxygenation, especially because these patients may not be able to generate sufficient tidal volumes on their own due to poor lung compliance.

252
Q

What ventilation strategy may be necessary for patients with RLD during anesthesia?
A) Lower inspiratory pressures to prevent lung damage
B) Increased inspiratory pressures to overcome decreased lung compliance
C) Use of positive end-expiratory pressure (PEEP) to reduce tidal volumes
D) Prolonged expiration times to increase carbon dioxide elimination

A

Correct Answer: B) Increased inspiratory pressures to overcome decreased lung compliance

Rationale: Due to the decreased lung compliance associated with RLD, increased inspiratory pressures may be necessary during mechanical ventilation to ensure adequate gas exchange.

253
Q

How does RLD contribute to perioperative risk?
A) By increasing the risk of fluid overload
B) By decreasing the risk of pulmonary complications
C) By contributing to the risk of perioperative pulmonary complications
D) By mitigating the effects of anesthesia on respiratory function

A

Correct Answer: C) By contributing to the risk of perioperative pulmonary complications

Rationale: RLD increases the risk of perioperative pulmonary complications due to the limited reserve in lung function, which can be further compromised by the effects of anesthesia, surgical positioning, and the inflammatory response to surgery.

254
Q

Why might postoperative mechanical ventilation be necessary in patients with RLD?
A) To prevent the risk of postoperative nausea and vomiting
B) To compensate for the reduced need for oxygen post-surgery
C) Because patients often experience improved lung function immediately after surgery
D) Due to impaired pulmonary function that may not recover immediately after anesthesia

A

Correct Answer: D) Due to impaired pulmonary function that may not recover immediately after anesthesia

Rationale: Patients with RLD may require postoperative mechanical ventilation because their already impaired pulmonary function can be further compromised by anesthesia and surgery, and it may not recover immediately, necessitating continued ventilatory support.

255
Q

Which procedure has generally replaced rigid bronchoscopy for obtaining samples from the airways for diagnostic purposes?
A) Thoracotomy
B) Fiberoptic bronchoscopy
C) Percutaneous needle biopsy
D) Mediastinoscopy

A

Correct Answer: B) Fiberoptic bronchoscopy

Rationale: Fiberoptic bronchoscopy has largely replaced rigid bronchoscopy because it is less invasive and can be used for visualizing the airways, obtaining samples for culture, cytologic examination, and biopsy with less discomfort and risk to the patient.

256
Q

What is the incidence of pneumothorax following a transbronchial lung biopsy?
A) 1-2%
B) 5-10%
C) 15-20%
D) Over 25%

A

Correct Answer: B) 5-10%

Rationale: Pneumothorax occurs in approximately 5-10% of patients after a transbronchial lung biopsy, which is a known risk of the procedure due to the potential for puncturing the lung tissue.

10-20% after percutaneous needle biopsy of peripheral lung lesions

257
Q

What is a major contraindication for performing a pleural biopsy?
A) Hypertension
B) Coagulopathy
C) Diabetes Mellitus
D) Chronic obstructive pulmonary disease (COPD)

A

Correct Answer: B) Coagulopathy

Rationale: The presence of a coagulopathy is a major contraindication for pleural biopsy due to the increased risk of bleeding during and after the procedure.

258
Q

During mediastinoscopy, the mediastinoscope can exert pressure on which artery, causing potential circulatory compromise?
A) Left subclavian artery
B) Right innominate artery
C) Left common carotid artery
D) Pulmonary artery

A

Correct Answer: B) Right innominate artery

Rationale: During mediastinoscopy, the mediastinoscope can exert pressure on the right innominate artery, which can lead to a loss of pulses in the right arm and compromise of right carotid artery blood flow.

259
Q

What are some risks associated with mediastinoscopy?

A) Pneumothorax, mediastinal hemorrhage, venous air embolism, and recurrent laryngeal nerve (RLN) injury
B) Arterial hypertension, pneumonitis, tracheal rupture, and esophageal injury
C) Bronchospasm, pleural effusion, hemothorax, and thoracic duct injury
D) Myocardial infarction, pulmonary embolism, aortic dissection, and stroke

A

Correct Answer: A) Pneumothorax, mediastinal hemorrhage, venous air embolism, and recurrent laryngeal nerve (RLN) injury

Rationale: Mediastinoscopy, a surgical procedure to examine the mediastinum, carries risks including pneumothorax, bleeding within the mediastinum (mediastinal hemorrhage), venous air embolism, and injury to the recurrent laryngeal nerve which can affect voice and breathing.

Mediastinoscopy is performed under GA through a small transverse incision just above the suprasternal notch